Download as pdf or txt
Download as pdf or txt
You are on page 1of 114

Endocrine NCLEX questions

Study online at https://quizlet.com/_5k4nly

7. A 67-year-old male client has been D.


complaining of sleeping more, increased Hyperparathyroidism is most common in
urination, anorexia, weakness, irritabil- older women and is characterized by
ity, depression, and bone pain that in- bone pain and weakness from excess
terferes with her going outdoors. Based parathyroid hormone (PTH). Clients also
on these assessment findings, nurse exhibit hypercaliuria-causing polyuria.
Richard would suspect which of the fol- While clients with diabetes mellitus and
lowing disorders? diabetes insipidus also have polyuria,
a. Diabetes mellitus they don't have bone pain and increased
b. Diabetes insipidus sleeping. Hypoparathyroidism is charac-
c. Hypoparathyroidism terized by urinary frequency rather than
d. Hyperparathyroidism polyuria.
B.
13. Early this morning, a female client Thyroid crisis usually occurs in the
had a subtotal thyroidectomy. During first 12 hours after thyroidectomy and
evening rounds, nurse Tina assesses causes exaggerated signs of hyperthy-
the client, who now has nausea, a tem- roidism, such as high fever, tachycar-
perature of 105° F (40.5° C), tachycar- dia, and extreme restlessness. Diabet-
dia, and extreme restlessness. What is ic ketoacidosis is more likely to pro-
the most likely cause of these signs? duce polyuria, polydipsia, and polypha-
a. Diabetic ketoacidosis gia; hypoglycemia, to produce weak-
b. Thyroid crisis ness, tremors, profuse perspiration, and
c. Hypoglycemia hunger. Tetany typically causes un-
d. Tetany controllable muscle spasms, stridor,
cyanosis, and possibly asphyxia.
25. During preoperative teaching for a
female client who will undergo subtotal D.
thyroidectomy, the nurse should include To prevent undue pressure on the sur-
which statement? gical incision after subtotal thyroidecto-
a. "The head of your bed must remain flat my, the nurse should advise the client
for 24 hours after surgery." to avoid hyperextending the neck. The
b. "You should avoid deep breathing and client may elevate the head of the bed as
coughing after surgery." desired and should perform deep breath-
c. "You won't be able to swallow for the ing and coughing to help prevent pneu-
first day or two." monia. Subtotal thyroidectomy doesn't
d. "You must avoid hyperextending your affect swallowing.
neck after surgery."

1 / 12
Endocrine NCLEX questions
Study online at https://quizlet.com/_5k4nly
In explaining the condition to a client,
a nurse would say that Cushing's syn-
drome is caused primarily by: D
Answers: Cushing's syndrome is caused by elevat-
A. Low levels of glucocorticoids ed levels of cortisol. Glucocorticoids tend
B. Excess secretion of sodium to cause this.
C. Autoimmunity in the pancreas
D. Elevated levels of cortisol
Which of the following symptoms is not
typical of Cushing's syndrome?
Answers: B
A. Osteoporosis Cushing's syndrome tends to produce
B. Weight loss rapid weight gain, not weight loss.
C. Diabetes
D. Mood instability
A client with Graves' disease experi-
ences a thyroid storm and has tachycar-
.D
dia and hypertension. What medication
Propylthiouracil is a commonly used
is most likely to be used?
medication for treating hyperthyroidism.
Answers:
Levofloxacin is an antibiotic, chloroth-
A. Levofloxcin
iazide is a diuretic, and Percocet a
B. Chlorothiazide
painkiller.
C. Percocet
D. Propylthiouracil
type 2 diabetes, hypertension and in-
creased risk of cardiovascular disease,
2. Grave's disease is: arthritis and colon polyps.
a. The most common cause of hypothy- 2. C: Grave's disease is an autoimmune
roidism disorder characterized by an enlarged
b. The most common cause of hyper- thyroid gland and overproduction of thy-
parathyroidism roid hormones producing symptoms of
c. The most common cause of hyperthy- hyperthyroidism such as rapid heart-
roidism beat, heat intolerance, agitation or irri-
d. The most common cause of adrenal tability, weight loss, and trouble sleeping.
insufficiency It usually presents in persons age 20
to 40 and it is much more common in
women than in men.

2 / 12
Endocrine NCLEX questions
Study online at https://quizlet.com/_5k4nly
3. C: Grave's ophthalmopathy is an in-
flammation of tissue behind the eye
causing the eyeballs to bulge. In addi-
3. Symptoms of Grave's ophthalmopathy tion to the above-mentioned symptoms,
include all of the following except: Grave's ophthalmopathy may cause
a. Bulging eyeballs pressure or pain in the eyes, double vi-
b. Dry, irritated eyes and puffy eyelids sion, and trouble moving the eyes. About
c. Cataracts one-quarter of persons with Grave's dis-
d. Light sensitivity ease develop Grave's ophthalmopathy.
The condition is frequently self-limit-
ing, resolving without treatment over the
course of a year or two.
5. D: Cushing's syndrome also may
5. All of the following are symptoms of cause fragile, thin skin prone to bruises
Cushing's syndrome except: and stretch marks on the abdomen and
a. Severe fatigue and weakness thighs as well as excessive thirst and
b. Hypertension and elevated blood glu- urination and mood changes such as de-
cose pression and anxiety. Women who suffer
c. A protruding hump between the shoul- from high levels of cortisol often have
ders irregular menstrual cycles or amenor-
d. Hair loss rhea and present with hair on their faces,
necks, chests, abdomens, and thighs.
6. D: Cushing's syndrome is a form of hy-
6. Which of the following conditions is
percortisolism. Risk factors for Cushing's
caused by long-term exposure to high
syndrome are obesity, diabetes, and hy-
levels of cortisol?
pertension. Cushing's syndrome is most
a. Addison's disease
frequently diagnosed in persons ages
b. Crohn's disease
20 to 50 who have characteristic round
c. Adrenal insufficiency
faces, upper body obesity, large necks,
d. Cushing's syndrome
and relatively thin limbs.
19. Endocrine disorders may be trig- 19. D: Endocrine function may be influ-
gered by all of the following except: enced by myriad factors. In addition to
a. Stress the above-mentioned, there is evidence
b. Infection that exposure to naturally occurring and
c. Chemicals in the food chain and envi- man-made endocrine disruptors such as
ronment tributyltin, certain bioaccumulating chlo-
d. overproduction of calcium deposits rinated compounds, and phytoestrogens

3 / 12
Endocrine NCLEX questions
Study online at https://quizlet.com/_5k4nly
is widespread and in susceptible individ-
uals, may trigger endocrine disorders.
The parathyroid glands play a major role
in regulating which substances?
A. Calcium and Phosphorus
B. Cholride and potassium a
C. Potassium and calcium
D. Sodium and potassium a. Calcium
and Phosphorus
The primary function of insuln is to:
A. Lower blood glucose levels
B. Produce melanin
a
C. regulate the bodys metabolic rate
D. stimulate release of digestive en-
zymes
a client is admitted to the hospital with
a medical DX of hyperthyroidism. When
taking a history which information would
be most significant?
A. edema, intolerance to cold, lethargy
b. peri-orbital edema, lethargy mask like d
face
c. weight loss, intolerance to cold, mus-
cle wasting
d. weight loss, intolerance to heat, ex-
ophthalmos
A nurse is preparing a diet plan for a 50yr
with simple goiter. Which of the following
should be included in the clients diet to
decrease the enlargement of he thyroid
gland? a
a. iodine
b. sodium
c. potassium
d. calcium
Replace intravenous fluids

4 / 12
Endocrine NCLEX questions
Study online at https://quizlet.com/_5k4nly
Induce shivering

Relieve respiratory distress

Administer a cooling blanket

Rationale: Thyroid storm (also called


thyroid crisis) is an extreme state of
hyperthyroidism that is rare today be-
A client presents to the emergency room
cause of improved diagnosis and treat-
with a history of Graves' disease. The
ment methods (Porth, 2005). When it
client reports having symptoms for a few
does occur, those affected are usually
days, but has not previously sought or
people with untreated hyperthyroidism
received any additional treatment. The
(most often Graves' disease) and people
client also reports having had a cold a
with hyperthyroidism who have experi-
few days back. Which of the following
enced a stressor, such as an infection,
interventions would be appropriate to im-
trauma. The rapid increase in metabol-
plement for this client, based on the his-
ic rate that results from the excessive
tory and current symptoms? Select all
TH causes the manifestations of thy-
that apply.
roid storm. The manifestations include
hyperthermia, with body temperatures
ranging from 102°F (39°C) to 106°F
Administer aspirin
(41°C); tachycardia; systolic hyperten-
sion; and gastrointestinal symptoms (ab-
Replace intravenous fluids
dominal pain, vomiting, diarrhea). Ag-
itation, restlessness, and tremors are
Induce shivering
common, progressing to confusion, psy-
chosis, delirium, and seizures. The mor-
Relieve respiratory distress
tality rate is high. Rapid treatment of thy-
roid storm is essential to preserve life.
Administer a cooling blanket
Treatment includes cooling without as-
pirin (which increases free TH) or in-
ducing shivering, replacing fluids, glu-
cose, and electrolytes, relieving respira-
tory distress, stabilizing cardiovascular
function, and reducing TH synthesis and
secretion. #1 is incorrect because cool-
ing happens without the use of aspirin.
All of the other choices are correct.
5 / 12
Endocrine NCLEX questions
Study online at https://quizlet.com/_5k4nly
A nursing student is studying for a test on
care of the client with endocrine disor-
ders. Which of the following statements
demonstrates an understanding of the
difference between hyperthyroidism and
hypothyroidism? Correct Answer: "Decreased renal blood
flow and glomerular filtration rate re-
duces the kidney's ability to excrete wa-
"Deficient amounts of TH cause abnor- ter, which may cause hyponatremia."
malities in lipid metabolism, with de- Rationale: # 1 is incorrect because defi-
creased serum cholesterol and triglyc- cient amounts of TH cause abnormalities
eride levels." in lipid metabolism with elevated serum
cholesterol and triglyceride levels. # 2 is
"Graves' disease is the most common incorrect because Graves' disease is the
cause of hypothyroidism." most common cause of hyperthyroidism,
not hypothyroidism. # 4 is incorrect be-
"Decreased renal blood flow and cause increased amounts of TH cause
glomerular filtration rate reduces the kid- an increase in cardiac output and periph-
ney's ability to excrete water, which may eral blood flow.
cause hyponatremia."

"Increased amounts of TH cause a de-


crease in cardiac output and peripheral
blood flow."
A nurse on a general medical-surgical
unit is caring for a client with Cushing's Mitotane is used to treat metastatic
syndrome. Which of the following state- adrenal cancer.
ments is correct about the medication Rationale: Mitotane directly suppress-
regimen for Cushing's syndrome? es activity of the adrenal cortex and de-
creases peripheral metabolism of corti-
costeroids. It is used to treat metastatic
Mitotane is used to treat metastatic adrenal cancer. # 2 is incorrect because
adrenal cancer. aminogluthimide may be administered to
clients with ectopic ACTH-secreting tu-
Aminogluthimide may be administered mors that cannot be surgically removed.
to clients with ectopic ACTH-secreting # 3 is incorrect because ketoconazole
tumors before surgery is performed. inhibits, not increases, cortisol synthesis
by the adrenal cortex. # 4 is incorrect
Ketoconazole increases cortisol synthe-
6 / 12
Endocrine NCLEX questions
Study online at https://quizlet.com/_5k4nly
sis by the adrenal cortex.
because somatostatin suppresses, not
Somatostatin analog increases ACTH increases, ACTH secretion.
secretion in some clients.
Assess respiratory rate, rhythm, depth,
A nurse on a surgical floor is caring for and effort.
a post-operative client who has just had Rationale: All of the above assessments
a subtotal thyroidectomy. Which of the have importance, but airway and breath-
following assessments should be com- ing in a client should always be ad-
pleted first on the client? dressed first when prioritizing care. As-
sess for signs of latent tetany due to
calcium deficiency, including tingling of
Assess for signs of tetany by checking toes, fingers, and lips; muscular twitch-
for Chvostek's and Trousseau's signs es; positive Chvostek's and Trousseau's
signs; and decreased serum calcium lev-
Assess dressing (if present) and the els. However, tetany may occur in 1 to
area under the client's neck and shoul- 7 days after thyroidectomy so # 1 is not
ders for drainage. the highest priority. Assessing for hemor-
rhage is always important, but the dan-
Administer analgesic pain medications ger of hemorrhage is greatest in the first
as ordered, and monitor their effective- 12 to 24 hours after surgery, and as this
ness. client is immediately post operative it is
not the main concern at this time. Pain
Assess respiratory rate, rhythm, depth, medication is important but according to
and effort. Maslow, pain is a psychosocial need to
be addressed after a physiologic need.
The nurse is caring for a client who
Maintain careful use of medical and sur-
is about to undergo an adrenalectomy.
gical asepsis when providing care and
Which of the following Preoperative in-
treatments.
terventions is most appropriate for this
Rationale: Use careful medical and sur-
client?
gical asepsis when providing care and
treatments since Cortisol excess in-
creases the risk of infection. # 2 is in-
correct. Nutrition should be addressed
Maintain careful use of medical and sur-
preoperatively. Request a dietary consul-
gical asepsis when providing care and
tation to discuss with the client about
treatments.
a diet high in vitamins and proteins. If

7 / 12
Endocrine NCLEX questions
Study online at https://quizlet.com/_5k4nly
Teach the client about a diet high in
sodium to correct any potential sodium hypokalemia exists, include foods high
imbalances preoperatively. in potassium. Glucocorticoid excess in-
creases catabolism. Vitamins and pro-
Explain to the client that electrolytes and teins are necessary for tissue repair and
glucose levels will be measured postop- wound healing following surgery. # 3 is
eratively. incorrect. Monitor the results of laborato-
ry tests of electrolytes and glucose lev-
Teach the client how to effectively cough els. Electrolyte and glucose imbalances
and deep breathe once surgery is com- are corrected
plete.
1) A
- An increased production of androgens
that accompanies a rise in cortisol levels
. The nurse assessing a female client
with Cushing's syndrome produces hir-
with Cushing's syndrome would expect
sutism and acne in women. Other clin-
to note which of the following?
ical findings of Cushing's syndrome in-
clude hypertension caused by sodium
a) hirsutism
retention, impaired glucose tolerance or
b) hypotension
diabetes mellitus caused by cortisol's
c) hypoglycemia
anti-insulin effect and ability to enhance
d) pallor
gluconeogenesis, and skin changes in-
cluding bruising and purplish red striae
caused by protein catabolism.
3) A
3. A nurse is preparing to perform an
- Excessive secretion of adrenocortical
assessment on a client being admitted
hormones results in water and sodi-
to the hospital with a diagnosis of Cush-
um reabsorption, causing fluid retention.
ing's syndrome. When performing the
Stretch marks (striae) are a common fea-
assessment, the nurse checks for which
ture and can result in a disturbed body
significant manifestation of the disorder?
image, but are not significant and do
not represent a life-threatening situation.
a) fluid retention
Goiter is not a manifestation of Cush-
b) stretch marks
ing's syndrome. Melanosis is a common
c) goiter
manifestation associated with Addison's
d) melanosis
disease.
. A client has been diagnosed with goi-
ter. The nurse looks for documentation
8 / 12
Endocrine NCLEX questions
Study online at https://quizlet.com/_5k4nly
of which of the following in the client's
6) C
medical record?
- An enlarged thyroid gland occurs in
goiter. Decreased wound healing, chron-
a) decreased wound healing
ic fatigue, and heart damage are not
b) chronic fatigue
specifically associated with this condi-
c) enlarged thyroid gland
tion.
d) heart damage
8) C
- Hypocalcemia can develop after thy-
roidectomy if the parathyroid glands
8. A nurse is caring for a client following are accidentally removed or trauma-
thyroidectomy and is monitoring for com- tized during surgery. If the client devel-
plications. Which of the following if noted ops numbness and tingling around the
in the client, would indicate a need for mouth or in the fingertips or toes, mus-
physician notification? cle spasms, or twitching, the physician
should be called immediately. A hoarse
a) surgical pain in the neck area or weak voice may occur temporarily if
b) voice hoarseness there has been unilateral injury to the la-
c) numbness and tingling around the ryngeal nerve during surgery. Pain is ex-
mouth pected in the postoperative period. Cal-
d) weakness of the voice cium gluconate ampules should be avail-
able at the bedside, and the client should
have a patent intravenous (IV) line in the
event that hypocalcemic tetany occurs
9. A nurse is monitoring a client for com-
plications following thyroidectomy. The
9) C
nurse notes that the client's voice is
Temporary hoarseness and a weak
very hoarse, and the client is concerned
voice may occur if there has been unilat-
about the hoarseness and asks the
eral injury to the laryngeal nerve during
nurse about it. The nurse makes which
surgery. If hoarseness or a weak voice
response to alleviate the client's con-
is present, the client is reassured that
cern?
the problem will probably subside in a
few days. Unnecessary talking is dis-
a) hoarseness and weak voice indicate
couraged to minimize hoarseness. The
permanent damage to the nerves
statements in options A, B, and D will not
b) this complication is expected
alleviate the client's concern.
c) this problem is temporary and will
probably subside in a few days

9 / 12
Endocrine NCLEX questions
Study online at https://quizlet.com/_5k4nly
d) it is best that you not talk at all until the
problem is further evaluated
16) D
- The client with hyperthyroidism is usu-
16. A nurse is caring for a client with
ally extremely hungry because of in-
hyperthyroidism and is instructing the
creased metabolism. The client should
client about dietary measures. The nurse
be instructed to consume a high-calorie
tells the client that it is important to eat
diet with six full meals a day. The client
foods that are:
should be instructed to eat foods that are
nutritious and contain ample amounts of
a) high in bulk and fiber
protein, carbohydrates, fats, and miner-
b) low in calories
als. Clients should be discouraged from
c) low in carbohydrates and fats
eating foods that increase peristalsis and
d) high in calories
thus result in diarrhea, such as highly
seasoned, bulky, and fibrous foods.
20. A husband of a client with graves' dis-
ease expresses concern regarding his
wife's health because during the past 3 20) A
months she has been experiencing ner- - Frequently, family and friends may re-
vousness, inability to concentrate even port that the client with Graves' disease
on trivial tasks, and outbursts of temper. has become more irritable or depressed.
On the basis of this information, which The signs and symptoms in the question
nursing diagnosis would the nurse iden- are supporting data for the nursing diag-
tify as appropriate for the client? nosis of Ineffective coping and are not re-
lated to options B, C, and D. The question
a) ineffective coping does not provide data to support options
b) disturbed sensory perception B, C, and D.
c) social isolation
d) grieving
27. A nurse provides instructions to a
client who is scheduled for a radioactive
iodine uptake test. Which statement by 27) D
the client indicates a need for further - The client undergoing a radioactive io-
instructions? dine uptake test needs to be reassured
that the amount of radioactive iodine
a) the test measures the rate of iodine used is very small, that it is not harmful
uptake by my thyroid gland
b) I will need to drink a small dose of
10 / 12
Endocrine NCLEX questions
Study online at https://quizlet.com/_5k4nly
radioactive iodine before the test
c) a 24 hour urine specimen will need to
be collected to measure iodine excretion to the client, and that the client will not be
d) I need to minimize close contact with radioactive. The other options are correct
others in my family for a period of 48 regarding this diagnostic test.
hours after the test because of the ra-
dioactivity in my system
29. A nurse is caring for a client with
29) D
Cushing's syndrome who demonstrates
- Physical changes in the client's ap-
withdrawn behavior. The nurse recog-
pearance can occur with Cushing's syn-
nizes that this client's behavior is likely
drome. Such changes include hirsutism,
related to which nursing diagnosis?
moon face, buffalo hump, acne, and stri-
ae. These changes cause a body image
a) deficient diversional activity
disturbance. Options A, B, and C are
b) powerlessness
not commonly associated with Cushing's
c) hopelessness
syndrome.
d) disturbed body image
A client is admitted for removal of a goi-
ter. Which nursing intervention should
receive priority during the post-operative
Answer B is correct. A goiter is hyper-
period?
plasia of the thyroid gland. Removal of
A. Maintaining fluid and electrolyte bal-
a goiter can result in laryngeal spasms
ance
and airway occlusion. The other answers
B. Assessing the client's airway
are lesser in priority.
C. Providing needed nutrition and fluids
D. Providing pain relief with narcotic
analgesics
Answer B is correct. A client with Cush-
A client with Cushing's syndrome should
ing's syndrome has adrenocortical hy-
be instructed to:
persecretion, so she retains sodium and
A. Avoid alcoholic beverages
water. The client may drink alcohol in
B. Limit the sodium in her diet
moderation, so answer A is incorrect,
C. Increase servings of dark green veg-
and there is no need to eat more green
etables
vegetables or limit protein, so answers C
D. Limit the amount of protein in her diet
and D are incorrect.
Answer D is correct. Laryngeal swelling
is not uncommon in clients following
a thyroidectomy. A tracheostomy tray
11 / 12
Endocrine NCLEX questions
Study online at https://quizlet.com/_5k4nly
should be kept available. The ventilator is
Which item should be kept at the bed-
not necessary, so answer A is incorrect.
side of a client who has just returned
The endotracheal tube is very difficult, if
from having a thyroidectomy?
not impossible, to intubate if swelling has
A. A padded tongue
already occurred, so answer B is incor-
B. An endotracheal tube
rect. The airway will do no good because
C. An airway
the swelling is in the trachea, so answer
D. A tracheostomy set
C is incorrect.

12 / 12
Lewis ch 48 Endocrine NCLEX
Study online at https://quizlet.com/_2phiql

ANS: A
Increased secretion of adrenocorti-
A 22-year-old patient is being seen in
cotropic hormone (ACTH) by the anterior
the clinic with increased secretion of the
pituitary gland will lead to an increase
anterior pituitary hormones. The nurse
in serum and urinary cortisol levels. An
would expect the laboratory results to
increase, rather than a decrease, in thy-
show
roxine level would be expected with in-
a. increased urinary cortisol.
creased secretion of thyroid stimulating
b. decreased serum thyroxine.
hormone (TSH) by the anterior pituitary.
c. elevated serum aldosterone levels.
Aldosterone and catecholamine levels
d. low urinary catecholamines excretion.
are not controlled by the anterior pitu-
itary.
Which statement by a 50-year-old fe-
ANS: D
male patient indicates to the nurse that
Difficulty in swallowing can occur with a
further assessment of thyroid function
goiter. Nocturia is associated with dis-
may be necessary?
eases such as diabetes mellitus, dia-
a. "I notice my breasts are tender lately."
betes insipidus, or chronic kidney dis-
b. "I am so thirsty that I drink all day long."
ease. Breast tenderness would occur
c. "I get up several times at night to uri-
with excessive gonadal hormone levels.
nate."
Thirst is a sign of disease such as dia-
d. "I feel a lump in my throat when I
betes.
swallow."
A 30-year-old patient seen in the emer-
gency department for severe headache
ANS: B
and acute confusion is found to have a
Elevated levels of antidiuretic hormone
serum sodium level of 118 mEq/L. The
will cause water retention and decrease
nurse will anticipate the need for which
serum sodium levels. The other tests
diagnostic test?
would not be helpful in determining pos-
a. Urinary 17-ketosteroids
sible causes of the patient's hyponatrem-
b. Antidiuretic hormone level
ia.
c. Growth hormone stimulation test
d. Adrenocorticotropic hormone level
Which question will provide the most
useful information to a nurse who is in-
ANS: C
terviewing a patient about a possible thy-
Because thyroid function affects meta-
roid disorder?
bolic rate, changes in weight may indi-
a. "What methods do you use to help
cope with stress?"
1/7
Lewis ch 48 Endocrine NCLEX
Study online at https://quizlet.com/_2phiql
b. "Have you experienced any blurring or
double vision?" cate hyperfunction or hypofunction of the
c. "Have you had a recent unplanned thyroid gland. Nocturia, visual difficulty,
weight gain or loss?" and changes in stress level are associ-
d. "Do you have to get up at night to ated with other endocrine disorders.
empty your bladder?"
A 29-year-old patient in the outpatient
clinic will be scheduled for blood cortisol
testing. Which instruction will the nurse
provide? ANS: C
a. "Avoid adding any salt to your foods Cortisol levels are usually drawn in the
for 24 hours before the test." morning, when levels are highest. The
b. "You will need to lie down for 30 min- other instructions would be given to pa-
utes before the blood is drawn." tients who were having other endocrine
c. "Come to the laboratory to have the testing.
blood drawn early in the morning."
d. "Do not have anything to eat or drink
before the blood test is obtained."
A 61-year-old female patient admitted
with pneumonia has a total serum calci- ANS: D
um level of 13.3 mg/dL (3.3 mmol/L). The Parathyroid hormone is the major con-
nurse will anticipate the need to teach troller of blood calcium levels. Although
the patient about testing for _____ lev- calcitonin secretion is a countermecha-
els. nism to parathyroid hormone, it does not
a. calcitonin play a major role in calcium balance. Cat-
b. catecholamine echolamine and thyroid hormone levels
c. thyroid hormone do not affect serum calcium level.
d. parathyroid hormone
During the physical examination of a
36-year-old female, the nurse finds that ANS: B
the patient's thyroid gland cannot be pal- The thyroid is frequently nonpalpable.
pated. The most appropriate action by The nurse should simply document the
the nurse is to finding. There is no need to notify the
a. palpate the patient's neck more health care provider immediately about a
deeply. normal finding. There is no indication for
b. document that the thyroid was nonpal- thyroid-stimulating hormone (TSH) test-
pable. ing unless there is evidence of thyroid
c. notify the health care provider imme-
2/7
Lewis ch 48 Endocrine NCLEX
Study online at https://quizlet.com/_2phiql
diately.
dysfunction. Deep palpation of the neck
d. teach the patient about thyroid hor-
is not appropriate.
mone testing.
Which laboratory value should the nurse
review to determine whether a patient's
ANS: C
hypothyroidism is caused by a problem
A low TSH level indicates that the pa-
with the anterior pituitary gland or with
tient's hypothyroidism is caused by de-
the thyroid gland?
creased anterior pituitary secretion of
a. Thyroxine (T4) level
TSH. Low T3 and T4 levels are not di-
b. Triiodothyronine (T3) level
agnostic of the primary cause of the
c. Thyroid-stimulating hormone (TSH)
hypothyroidism. TRH levels indicate the
level
function of the hypothalamus.
d. Thyrotropin-releasing hormone (TRH)
level
ANS: C
The nurse reviews a patient's glycosy- Glycosylated hemoglobin testing mea-
lated hemoglobin (Hb A1C) results to sures glucose control over the last 3
evaluate months. Glucose testing before/after a
a. fasting preprandial glucose levels. meal or random testing may reveal im-
b. glucose levels 2 hours after a meal. paired glucose tolerance and indicate
c. glucose control over the past 90 days. prediabetes, but it is not done on pa-
d. hypoglycemic episodes in the past 3 tients who already have a diagnosis of
months. diabetes. There is no test to evaluate for
hypoglycemic episodes in the past.
ANS: C
A 60-year-old patient is taking spirono-
Because aldosterone increases the ex-
lactone (Aldactone), a drug that blocks
cretion of potassium, a medication that
the action of aldosterone on the kidney,
blocks aldosterone will tend to cause hy-
for hypertension. The nurse will monitor
perkalemia. Aldosterone also promotes
for
the reabsorption of sodium and water in
a. increased serum sodium.
the renal tubules, so spironolactone will
b. decreased urinary output.
tend to cause increased urine output, a
c. elevated serum potassium.
decreased or normal serum sodium lev-
d. evidence of fluid overload.
el, and signs of dehydration.
A 40-year-old male patient has been
ANS: B
newly diagnosed with type 2 diabetes
When dealing with a patient with a chron-
mellitus. Which information about the pa-
ic condition such as diabetes, identifi-
tient will be most useful to the nurse
3/7
Lewis ch 48 Endocrine NCLEX
Study online at https://quizlet.com/_2phiql
who is helping the patient develop strate- cation of the patient's values and be-
gies for successful adaptation to this dis- liefs can assist the health care team
ease? in choosing strategies for successful
a. Ideal weight lifestyle change. The other information
b. Value system also will be useful, but is not as important
c. Activity level in developing an individualized plan for
d. Visual changes the necessary lifestyle changes.
ANS: D
Hypoglycemia is induced during the
An 18-year-old male patient with a small
growth hormone stimulation test, and the
stature is scheduled for a growth hor-
nurse should be ready to administer 50%
mone stimulation test. In preparation for
dextrose immediately. Regular insulin is
the test, the nurse will obtain
used to induce hypoglycemia (glargine is
a. ice in a basin.
never given IV). The patient does not re-
b. glargine insulin.
quire cardiac monitoring during the test.
c. a cardiac monitor.
Although blood samples for some tests
d. 50% dextrose solution.
must be kept on ice, this is not true for
the growth hormone stimulation test.
The nurse will teach a patient to plan to
minimize physical and emotional stress
while the patient is undergoing ANS: C
a. a water deprivation test. Physical and emotional stress can affect
b. testing for serum T3 and T4 levels. the results of the free cortisol test. The
c. a 24-hour urine test for free cortisol. other tests are not impacted by stress.
d. a radioactive iodine (I-131) uptake
test.
A nurse will teach a patient who is sched-
uled to complete a 24-hour urine collec-
ANS: B
tion for 17-ketosteroids to
The specimen must be kept on ice or re-
a. insert and maintain a retention
frigerated until the collection is finished.
catheter.
Voided or catheterized specimens are
b. keep the specimen refrigerated or on
acceptable for the test. The initial voided
ice.
specimen is discarded. There is no fluid
c. drink at least 3 L of fluid during the 24
intake requirement for the 24-hour col-
hours.
lection.
d. void and save that specimen to start
the collection.

4/7
Lewis ch 48 Endocrine NCLEX
Study online at https://quizlet.com/_2phiql
Which additional information will the
nurse need to consider when reviewing ANS: C
the laboratory results for a patient's total Part of the total calcium is bound to al-
calcium level? bumin so hypoalbuminemia can lead to
a. The blood glucose is elevated. misinterpretation of calcium levels. The
b. The phosphate level is normal. other laboratory values will not affect to-
c. The serum albumin level is low. tal calcium interpretation.
d. The magnesium level is normal.
A 44-year-old patient is admitted with
tetany. Which laboratory value should
ANS: C
the nurse monitor?
Tetany is associated with hypocalcemia.
a. Total protein
The other values would not be useful for
b. Blood glucose
this patient.
c. Ionized calcium
d. Serum phosphate
Which information about a 30-year-old
patient who is scheduled for an oral glu-
cose tolerance test should be reported
to the health care provider before start-
ing the test? ANS: B
a. The patient reports having occasional Corticosteroids can affect blood glucose
orthostatic dizziness. results. The other information will be pro-
b. The patient takes oral corticosteroids vided to the health care provider but will
for rheumatoid arthritis. not affect the test results.
c. The patient has had a 10-pound
weight gain in the last month.
d. The patient drank several glasses of
water an hour previously.
Which action by a new registered nurse
(RN) caring for a patient with a goiter
ANS: B
and possible hyperthyroidism indicates
Palpation can cause the release of thy-
that the charge nurse needs to do more
roid hormones in a patient with an en-
teaching?
larged thyroid and should be avoided.
a. The RN checks the blood pressure on
The other actions by the new RN are ap-
both arms.
propriate when caring for a patient with
b. The RN palpates the neck thoroughly
an enlarged thyroid.
to check thyroid size.
c. The RN lowers the thermostat to de-
5/7
Lewis ch 48 Endocrine NCLEX
Study online at https://quizlet.com/_2phiql
crease the temperature in the room.
d. The RN orders nonmedicated eye
drops to lubricate the patient's bulging
eyes.
The nurse is caring for a 45-year-old
male patient during a water deprivation
test. Which finding is most important for
the nurse to communicate to the health ANS: B
care provider? A drop in the weight of more than 2
a. The patient complains of intense thirst. kg indicates severe dehydration, and the
b. The patient has a 5-lb (2.3 kg) weight test should be discontinued. The other
loss. assessment data are not unusual with
c. The patient's urine osmolality does not this test.
increase.
d. The patient feels dizzy when sitting on
the edge of the bed.
A 35-year-old female patient with a pos-
sible pituitary adenoma is scheduled for
a computed tomography (CT) scan with ANS: B
contrast media. Which patient informa- Because the usual contrast media is io-
tion is most important for the nurse to dine-based, the health care provider will
communicate to the health care provider need to know about the allergy before
before the test? the CT scan. The other findings are com-
a. Bilateral poor peripheral vision mon with any mass in the brain such as
b. Allergies to iodine and shellfish a pituitary adenoma.
c. Recent weight loss of 20 pounds
d. Complaint of ongoing headaches
The nurse is caring for a 63-year-old
with a possible pituitary tumor who is
ANS: A
scheduled for a computed tomography
Because contrast media may cause
(CT) scan with contrast. Which informa-
acute kidney injury in patients with poor
tion about the patient is most important
renal function, the health care provider
to discuss with the health care provider
will need to prescribe therapies such as
before the test?
IV fluids to prevent this complication. The
a. History of renal insufficiency
other findings are consistent with the pa-
b. Complains of chronic headache
tient's diagnosis of a pituitary tumor.
c. Recent bilateral visual field loss
d. Blood glucose level of 134 mg/dL
6/7
Lewis ch 48 Endocrine NCLEX
Study online at https://quizlet.com/_2phiql
Which statements will the nurse include
when teaching a patient who is sched-
uled for oral glucose tolerance testing in
ANS: A, C, E
the outpatient clinic (select all that ap-
Smoking may affect the results of oral
ply)?
glucose tolerance tests. Blood samples
a. "You will need to avoid smoking before
are obtained at baseline and at 30, 60,
the test."
and 120 minutes. Accuracy requires that
b. "Exercise should be avoided until the
the patient be fasting before the test. The
testing is complete."
patient should consume at least 1500
c. "Several blood samples will be ob-
calories/day for 3 days before the test.
tained during the testing."
The patient should be ambulatory and
d. "You should follow a low-calorie diet
active for accurate test results.
the day before the test."
e. "The test requires that you fast for at
least 8 hours before testing."

7/7
endocrine NCLEX Questions
Study online at https://quizlet.com/_96twcd

1. Check for signs of bleeding.


2. Administer calcium gluconate.
3. Notify the registered nurse immediate-
A nurse is collecting data regarding a
ly.
client after a thyroidectomy and notes
*4. Reassure the client that this is usually
that the client has developed hoarse-
a temporary condition.*
ness and a weak voice. Which nursing
*rationale* Weakness and hoarseness of
action is appropriate?
the voice can occur as a result of trau-
1. Check for signs of bleeding.
ma of the laryngeal nerve. If this devel-
2. Administer calcium gluconate.
ops, the client should be reassured that
3. Notify the registered nurse immediate-
the problem will subside in a few days.
ly.
Unnecessary talking should be discour-
4. Reassure the client that this is usually
aged. It is not necessary to notify the reg-
a temporary condition.
istered nurse immediately. These signs
do not indicate bleeding or the need to
administer calcium gluconate.
*1. "I can eat foods that contain potassi-
um."*
A nurse is reviewing discharge teaching
2. "I will need to limit the amount of pro-
with a client who has Cushing's syn-
tein in my diet."
drome. Which statement by the client in-
3. "I am fortunate that I can eat all the
dicates that the instructions related to
salty foods I enjoy."
dietary management were understood?
4. "I am fortunate that I do not need to
1. "I can eat foods that contain potassi-
follow any special diet."
um."
*rationale* A diet that is low in calo-
2. "I will need to limit the amount of pro-
ries, carbohydrates, and sodium but am-
tein in my diet."
ple in protein and potassium content is
3. "I am fortunate that I can eat all the
encouraged for a client with Cushing's
salty foods I enjoy."
syndrome. Such a diet promotes weight
4. "I am fortunate that I do not need to
loss, the reduction of edema and hyper-
follow any special diet."
tension, the control of hypokalemia, and
the rebuilding of wasted tissue.
1. "Cushing's disease is characterized by
A nursing instructor asks a student to de-
an oversecretion of insulin."
scribe the pathophysiology that occurs in
*2. "Cushing's disease is characterized
Cushing's disease. Which statement by
by an oversecretion of glucocorticoid
the student indicates an accurate under-
hormones."*
standing of this disorder?
3. "Cushing's disease is characterized by
1 / 20
endocrine NCLEX Questions
Study online at https://quizlet.com/_96twcd
an undersecretion of corticotropic hor-
1. "Cushing's disease is characterized mones."
by an oversecretion of insulin." 4. "Cushing's disease is characterized
2. "Cushing's disease is characterized by an undersecretion of glucocorticoid
by an oversecretion of glucocorticoid hormones."
hormones." *rationale* Cushing's syndrome is char-
3. "Cushing's disease is characterized acterized by an oversecretion of glu-
by an undersecretion of corticotropic cocorticoid hormones. Addison's dis-
hormones." ease is characterized by the failure of
4. "Cushing's disease is characterized the adrenal cortex to produce and se-
by an undersecretion of glucocorticoid crete adrenocortical hormones. Options
hormones." 1 and 4 are inaccurate regarding Cush-
ing's syndrome.
1. Provide a cool environment for the
client.
2. Instruct the client to consume a
high-fat diet.
*3. Instruct the client about thyroid re-
A nurse would expect to note which in- placement therapy.*
terventions in the plan of care for a client *4. Encourage the client to consume flu-
with hypothyroidism? *Select all that ap- ids and high-fiber foods in the diet.*
ply.* *5. Instruct the client to contact the health
1. Provide a cool environment for the care provider if episodes of chest pain
client. occur.*
2. Instruct the client to consume a 6. Inform the client that iodine prepara-
high-fat diet. tions will be prescribed to treat the disor-
3. Instruct the client about thyroid re- der.
placement therapy. *rationale* The clinical manifestations
4. Encourage the client to consume flu- of hypothyroidism are the result of de-
ids and high-fiber foods in the diet. creased metabolism from low levels
5. Instruct the client to contact the health of thyroid hormone. Interventions are
care provider if episodes of chest pain aimed at replacement of the hormones
occur. and providing measures to support the
6. Inform the client that iodine prepara- signs and symptoms related to a de-
creased metabolism. The nurse encour-
ages the client to consume a well-bal-
anced diet that is low in fat for weight re-
duction and high in fluids and high-fiber
foods to prevent constipation. The client
2 / 20
endocrine NCLEX Questions
Study online at https://quizlet.com/_96twcd
often has cold intolerance and requires a
warm environment. The client would no-
tify the health care provider if chest pain
occurs since it could be an indication
tions will be prescribed to treat the dis- of overreplacement of thyroid hormone.
order. Iodine preparations are used to treat
hyperthyroidism. These medications de-
crease blood flow through the thyroid
gland and reduce the production and re-
lease of thyroid hormone.
1. Weigh the client.
2. Test the client's urine for glucose.
*3. Monitor the client's blood pressure.*
Which nursing action would be appro-
4. Palpate the client's skin to determine
priate to implement when a client has a
warmth.
diagnosis of pheochromocytoma?
*rationale* Hypertension is the ma-
1. Weigh the client.
jor symptom that is associated with
2. Test the client's urine for glucose.
pheochromocytoma. The blood pressure
3. Monitor the client's blood pressure.
status is monitored by taking the client's
4. Palpate the client's skin to determine
blood pressure. Glycosuria, weight loss,
warmth.
and diaphoresis are also clinical mani-
festations of pheochromocytoma, but hy-
pertension is the major symptom.
*1. Vital signs*
2. Intake and output
A nurse is caring for a client with 3. Blood urea nitrogen (BUN) level
pheochromocytoma. The client is sched- 4. Urine for glucose and acetone
uled for an adrenalectomy. During the *rationale* Hypertension is the hallmark
preoperative period, the priority nursing of pheochromocytoma. Severe hyper-
action would be to monitor the: tension can precipitate a brain attack
1. Vital signs (stroke) or sudden blindness. Although
2. Intake and output all of the options are accurate nursing in-
3. Blood urea nitrogen (BUN) level terventions for the client with pheochro-
4. Urine for glucose and acetone mocytoma, the priority nursing action is
to monitor the vital signs, particularly the
blood pressure.
A nurse is caring for a client with 1. Crackers with cheese and tea
pheochromocytoma. The client asks for *2. Graham crackers and warm milk*
3 / 20
endocrine NCLEX Questions
Study online at https://quizlet.com/_96twcd
3. Toast with peanut butter and cocoa
a snack and something warm to drink.
4. Vanilla wafers and coffee with cream
The appropriate choice for this client to
and sugar
meet nutritional needs would be which of
*rationale* The client with pheochromo-
the following?
cytoma needs to be provided with a diet
1. Crackers with cheese and tea
that is high in vitamins, minerals, and
2. Graham crackers and warm milk
calories. Of particular importance is that
3. Toast with peanut butter and cocoa
food or beverages that contain caffeine
4. Vanilla wafers and coffee with cream
(e.g., chocolate, coffee, tea, and cola)
and sugar
are prohibited.
1. A urinary output of 50 mL/hr
2. A coagulation time of 5 minutes
*3. Congestion heard on auscultation of
the lungs*
4. A blood urea nitrogen (BUN) level of
20 mg/dL
A nurse is caring for a client with
*rationale* The complications associat-
pheochromocytoma. Which data would
ed with pheochromocytoma include hy-
indicate a potential complication associ-
pertensive retinopathy and nephropa-
ated with this disorder?
thy, myocarditis, congestive heart failure
1. A urinary output of 50 mL/hr
(CHF), increased platelet aggregation,
2. A coagulation time of 5 minutes
and stroke. Death can occur from shock,
3. Congestion heard on auscultation of
stroke, renal failure, dysrhythmias, or
the lungs
dissecting aortic aneurysm. Congestion
4. A blood urea nitrogen (BUN) level of
heard on auscultation of the lungs is in-
20 mg/dL
dicative of CHF. A urinary output of 50
mL/hr is an appropriate output; the nurse
would become concerned if the output
were less than 30 mL/hr. A coagulation
time of 5 minutes is normal. A BUN level
of 20 mg/dL is a normal finding.
1. Bradycardia
A nurse is caring for a client after thy- *2. Hypotension*
roidectomy and monitoring for signs of 3. Constipation
thyroid storm. The nurse understands 4. Hypothermia
that which of the following is a manifes- *rationale* Clinical manifestations asso-
tation associated with this disorder? ciated with thyroid storm include a fever
as high as 106° F (41.1° C), severe

4 / 20
endocrine NCLEX Questions
Study online at https://quizlet.com/_96twcd
tachycardia, profuse diarrhea, extreme
vasodilation, hypotension, atrial fibrilla-
1. Bradycardia
tion, hyperreflexia, abdominal pain, di-
2. Hypotension
arrhea, and dehydration. With this dis-
3. Constipation
order, the client's condition can rapid-
4. Hypothermia
ly progress to coma and cardiovascular
collapse.
1. Lower the head of the bed.
*2. Test the drainage for glucose.*
3. Obtain a culture of the drainage.
4. Continue to observe the drainage.
*rationale* After hypophysectomy, the
When caring for a client who is hav-
client should be monitored for rhin-
ing clear drainage from his nares after
orrhea, which could indicate a cere-
transsphenoidal hypophysectomy, which
brospinal fluid (CSF) leak. If this occurs,
action by the nurse is appropriate?
the drainage should be collected and
1. Lower the head of the bed.
tested for glucose, indicating the pres-
2. Test the drainage for glucose.
ence of CSF. The head of the bed should
3. Obtain a culture of the drainage.
not be lowered to prevent increased in-
4. Continue to observe the drainage.
tracranial pressure. Clear nasal drainage
would not indicate the need for a culture.
Continuing to observe the drainage with-
out taking action could result in a serious
complication.
1. Edema
2. Obesity
3. Hirsutism
Which clinical manifestation should the
*4. Hypotension*
nurse expect to note when assessing a
*rationale* Common manifestations of
client with Addison's disease?
Addison's disease include postural hy-
1. Edema
potension from fluid loss, syncope, mus-
2. Obesity
cle weakness, anorexia, nausea, vomit-
3. Hirsutism
ing, abdominal cramps, weight loss, de-
4. Hypotension
pression, and irritability. The manifesta-
tions in options 1, 2, and 3 are not asso-
ciated with Addison's disease.
1. Provide a high-fiber diet.
*2. Provide a restful environment.*
5 / 20
endocrine NCLEX Questions
Study online at https://quizlet.com/_96twcd
3. Provide three small meals per day.
4. Provide the client with extra blankets.
*rationale* Because of the hypermeta-
What would the nurse anticipate being bolic state, the client with Graves' dis-
included in the plan of care for a client ease needs to be provided with an en-
who has been diagnosed with Graves' vironment that is restful both physically
disease? and mentally. Six full meals a day that
1. Provide a high-fiber diet. are well balanced and high in calories
2. Provide a restful environment. are required, because of the accelerated
3. Provide three small meals per day. metabolic rate. Foods that increase peri-
4. Provide the client with extra blankets. stalsis (e.g., high-fiber foods) need to be
avoided. These clients suffer from heat
intolerance and require a cool environ-
ment.
*1. "I had a radionuclide test done 3 days
ago."*
Which statement by the client would
2. "When I exercise I sweat more than
cause the nurse to suspect that the thy-
normal."
roid test results drawn on the client this
3. "I drank some water before the blood
morning may be inaccurate?
was drawn."
1. "I had a radionuclide test done 3 days
4. "That hamburger I ate before the test
ago."
sure tasted good."
2. "When I exercise I sweat more than
*rationale* Option 1 indicates that a re-
normal."
cent radionuclide scan had been per-
3. "I drank some water before the blood
formed. Recent radionuclide scans per-
was drawn."
formed before the test can affect thyroid
4. "That hamburger I ate before the test
laboratory results. No food, fluid, or activ-
sure tasted good."
ity restrictions are required for this test,
so options 2, 3, and 4 are incorrect.
1. Low-protein diet
A nurse is preparing to provide instruc- 2. Low-sodium diet
tions to a client with Addison's disease *3. High-sodium diet*
regarding diet therapy. The nurse under- 4. Low-carbohydrate diet
stands that which of the following diets *rationale* A high-sodium, high-complex
would likely be prescribed for this client? carbohydrate, and high-protein diet will
1. Low-protein diet be prescribed for the client with Addi-
2. Low-sodium diet son's disease. To prevent excess fluid
and sodium loss, the client is instructed

6 / 20
endocrine NCLEX Questions
Study online at https://quizlet.com/_96twcd
to maintain an adequate salt intake of up
to 8 g of sodium daily and to increase salt
3. High-sodium diet
intake during hot weather, before stren-
4. Low-carbohydrate diet
uous exercise, and in response to fever,
vomiting, or diarrhea.
1. Hair will need to be shaved.
A nurse caring for a client scheduled 2. Deep breathing and coughing will be
for a transsphenoidal hypophysectomy needed after surgery.
to remove a tumor in the pituitary gland *3. Toothbrushing will not be permitted
assists to develop a plan of care for for at least 2 weeks following surgery.*
the client. The nurse suggests including 4. Spinal anesthesia is used.
which specific information in the preop- *rationale* Based on the location of the
erative teaching plan? surgical procedure, spinal anesthesia
1. Hair will need to be shaved. would not be used. In addition, the hair
2. Deep breathing and coughing will be would not be shaved. Although coughing
needed after surgery. and deep breathing are important, spe-
3. Toothbrushing will not be permitted for cific to this procedure is avoiding tooth-
at least 2 weeks following surgery. brushing to prevent disruption of the sur-
4. Spinal anesthesia is used. gical site. Also, coughing may disrupt the
surgical site.
*1. Signs and symptoms of hypothy-
A health care provider has prescribed
roidism*
propylthiouracil (PTU) for a client with
2. Signs and symptoms of hyper-
hyperthyroidism, and the nurse assists
glycemia
in developing a plan of care for the client.
3. Relief of pain
A priority nursing measure to be includ-
4. Signs of renal toxicity
ed in the plan regarding this medication
*rationale* Excessive dosing with propy-
is to monitor the client for:
lthiouracil may convert the client from a
1. Signs and symptoms of hypothy-
hyperthyroid state to a hypothyroid state.
roidism
If this occurs, the dosage should be re-
2. Signs and symptoms of hyper-
duced. Temporary administration of thy-
glycemia
roid hormone may be required. Propylth-
3. Relief of pain
iouracil is not used for pain and does not
4. Signs of renal toxicity
cause hyperglycemia or renal toxicity.
1. Administering methimazole (Tapa-
zole) every 8 hours
2. Lubricating the eyes with tap water
every 2 to 4 hours
7 / 20
endocrine NCLEX Questions
Study online at https://quizlet.com/_96twcd
3. Instructing the client to avoid straining
or heavy lifting
*4. Obtaining dark glasses for the client*
*rationale* Because photophobia (light
intolerance) accompanies this disorder,
A client with Graves' disease has exoph- dark glasses are helpful in alleviating the
thalmos and is experiencing photopho- symptom. Medical therapy for Graves'
bia. Which intervention would best assist disease does not help alleviate the clin-
the client with this problem? ical manifestation of exophthalmos. Oth-
1. Administering methimazole (Tapa- er interventions may be used to relieve
zole) every 8 hours the drying that occurs from not being
2. Lubricating the eyes with tap water able to completely close the eyes; how-
every 2 to 4 hours ever, the question is asking what the
3. Instructing the client to avoid straining nurse can do for photophobia. Tap water,
or heavy lifting which is hypotonic, could actually cause
4. Obtaining dark glasses for the client more swelling to the eye because it could
pull fluid into the interstitial space. In ad-
dition, the client is at risk for developing
an eye infection because the solution is
not sterile. There is no need to prevent
straining with exophthalmos.
1. Dehydration
2. Infection
3. Urinary retention
*4. Bleeding*
The nurse caring for a client who has *rationale* Hemorrhage is one of the
had a subtotal thyroidectomy reviews the most severe complications that can oc-
plan of care and determines which prob- cur following thyroidectomy. The nurse
lem is the priority for this client in the must frequently check the neck dressing
immediate postoperative period? for bleeding and monitor vital signs to
1. Dehydration detect early signs of hemorrhage, which
2. Infection could lead to shock. T3 and T4 do not
3. Urinary retention regulate fluid volumes in the body. Infec-
4. Bleeding tion is a concern for any postoperative
client but is not the priority in the immedi-
ate postoperative period. Urinary reten-
tion can occur in postoperative clients
as a result of medication and anesthesia

8 / 20
endocrine NCLEX Questions
Study online at https://quizlet.com/_96twcd
but is not the priority from the options
provided.
1. Auscultation of lung sounds
*2. Inspection of facial features*
A nurse is collecting data on a client
3. Percussion of the thyroid gland
admitted to the hospital with a diagno-
4. Palpation of the adrenal glands
sis of myxedema. Which data collection
*rationale* Inspection of facial features
technique will provide data necessary to
will reveal the characteristic coarse fea-
support the admitting diagnosis?
tures, presence of edema around the
1. Auscultation of lung sounds
eyes and face, and a blank expression
2. Inspection of facial features
that are characteristic of myxedema. The
3. Percussion of the thyroid gland
techniques in the remaining options will
4. Palpation of the adrenal glands
not reveal any data that would support
the diagnosis of myxedema.
1. Nervousness
*2. Infection*
3. Concern about appearance
A client with Cushing's disease is be- 4. Inability to care for self
ing admitted to the hospital after a stab *rationale* The client with a stab wound
wound to the abdomen. The nurse plans has a break in the body's first line of
care and places highest priority on which defense against infection. The client with
potential problem? Cushing's disease is at great risk for in-
1. Nervousness fection because of excess cortisol secre-
2. Infection tion and subsequent impaired antibody
3. Concern about appearance function and decreased proliferation of
4. Inability to care for self lymphocytes. The client may also have
a potential for the problems listed in the
other options but these are not the high-
est priority at this time.
*1. Monitoring daily weight*
A nurse notes in the medical record that *2. Monitoring intake and output*
a client with Cushing's syndrome is ex- 3. Maintaining a low-potassium diet
periencing fluid overload. Which inter- *4. Monitoring extremities for edema*
ventions should be included in the plan *5. Maintaining a low-sodium diet*
of care? *Select all that apply.* *rationale* The client with Cushing's
1. Monitoring daily weight syndrome experiencing fluid overload
2. Monitoring intake and output should be maintained on a high-potas-
sium and low-sodium diet. Decreased
9 / 20
endocrine NCLEX Questions
Study online at https://quizlet.com/_96twcd
sodium intake decreases renal retention
3. Maintaining a low-potassium diet of sodium and water. Monitoring weight,
4. Monitoring extremities for edema intake, output, and extremities for edema
5. Maintaining a low-sodium diet are all appropriate interventions for such
a nursing diagnosis.
1. Encouraging the client's expression of
A nursing student notes in the medical feelings
record that a client with Cushing's syn- 2. Evaluating the client's understanding
drome is experiencing body image dis- of the disease process
turbances. The need for additional edu- 3. Encouraging family members to share
cation regarding this problem is identi- their feelings about the disease process
fied when the nursing student suggests *4. Evaluating the client's understanding
which nursing intervention? that the body changes need to be dealt
1. Encouraging the client's expression of with*
feelings *rationale* Evaluating the client's under-
2. Evaluating the client's understanding standing that the body changes that oc-
of the disease process cur in this disorder need to be dealt with
3. Encouraging family members to share is an inappropriate nursing intervention.
their feelings about the disease process This option does not address the client's
4. Evaluating the client's understanding feelings. Options 1, 2, and 3 are appro-
that the body changes need to be dealt priate because they address the client
with and family feelings regarding the disor-
der.
A nurse is caring for a client following 1. Double vision
an adrenalectomy and is monitoring for *2. Hypotension*
signs of adrenal insufficiency. Which of *3. Mental status changes*
the following, if noted in the client, in- *4. Weakness*
dicates signs and symptoms related to *5. Fever*
adrenal insufficiency? *Select all that ap- *rationale* The nurse should be alert to
ply.* signs and symptoms of adrenal insuffi-
1. Double vision ciency in a client following adrenalecto-
2. Hypotension my. These signs and symptoms include
3. Mental status changes weakness, hypotension, fever, and men-
4. Weakness tal status changes. Double vision is gen-
5. Fever erally not associated with this condition.
1. "Taking my medications exactly as
prescribed is essential."
*2. "I need to read the labels on
10 / 20
endocrine NCLEX Questions
Study online at https://quizlet.com/_96twcd
any over-the-counter medications I pur-
chase."*
A nurse is reinforcing home care in-
3. "My family needs to be familiar with
structions to a client with a diagnosis of
the signs and symptoms of hypoadren-
Cushing's syndrome. Which statement
alism."
reflects a need for further client educa-
4. "I could experience the signs and
tion?
symptoms of hyperadrenalism because
1. "Taking my medications exactly as
of Cushing's."
prescribed is essential."
*rationale* The client with Cushing's syn-
2. "I need to read the labels on
drome should be instructed to take the
any over-the-counter medications I pur-
medications exactly as prescribed. The
chase."
nurse should emphasize the importance
3. "My family needs to be familiar with
of continuing medications, consulting
the signs and symptoms of hypoadrena-
with the health care provider before pur-
lism."
chasing any over-the-counter medica-
4. "I could experience the signs and
tions, and maintaining regular follow-up
symptoms of hyperadrenalism because
care. The nurse should also instruct the
of Cushing's."
client in the signs and symptoms of both
hypoadrenalism and hyperadrenalism.
A nurse is reviewing a plan of care for a *1. Encouraging fluid intake of at least
client with Addison's disease. The nurse 3000 mL/day*
notes that the client is at risk for dehydra- 2. Encouraging an intake of low-protein
tion and suggests nursing interventions foods
that will prevent this occurrence. Which *3. Monitoring for changes in mental sta-
nursing intervention is an appropriate tus*
component of the plan of care? *Select *4. Monitoring intake and output*
all that apply.* 5. Maintaining a low-sodium diet
1. Encouraging fluid intake of at least *rationale* The client at risk for defi-
3000 mL/day cient fluid volume should be encouraged
2. Encouraging an intake of low-protein to eat regular meals and snacks and
foods to increase the intake of sodium, pro-
3. Monitoring for changes in mental sta- tein, and complex carbohydrates. Oral
tus replacement of sodium losses is nec-
4. Monitoring intake and output essary, and maintenance of adequate
5. Maintaining a low-sodium diet blood glucose levels is required.
1. Maintain a supine position.
2. Encourage coughing and deep breath-
ing exercises.

11 / 20
endocrine NCLEX Questions
Study online at https://quizlet.com/_96twcd
*3. Monitor neck circumference frequent-
ly.*
4. Maintain a pressure dressing on the
A nurse reviews a plan of care for a post-
operative site.
operative client following a thyroidecto-
*rationale* Following a thyroidectomy,
my and notes that the client is at risk for
the client should be placed in an up-
breathing difficulty. Which of the follow-
right position to facilitate air exchange.
ing nursing interventions will the nurse
The nurse should assist the client with
suggest to include in the plan of care?
deep breathing exercises, but coughing
1. Maintain a supine position.
is minimized to prevent tissue damage
2. Encourage coughing and deep
and stress to the incision. A pressure
breathing exercises.
dressing is not placed on the operative
3. Monitor neck circumference frequent-
site because it could affect breathing.
ly.
The nurse should monitor the dressing
4. Maintain a pressure dressing on the
closely and should loosen the dressing if
operative site.
necessary. Neck circumference is moni-
tored at least every 4 hours to assess for
postoperative edema.
*1. Tingling around the mouth*
2. Negative Chvostek's sign
A nurse is monitoring a client following a 3. Flaccid paralysis
thyroidectomy for signs of hypocalcemia. 4. Bradycardia
Which of the following signs, if noted in *rationale* Following a thyroidectomy,
the client, likely indicates the presence the nurse assesses the client for signs
of hypocalcemia? of hypocalcemia and tetany. Early signs
1. Tingling around the mouth include tingling around the mouth and
2. Negative Chvostek's sign fingertips, muscle twitching or spasms,
3. Flaccid paralysis palpitations or dysrhythmias, and posi-
4. Bradycardia tive Chvostek's and Trousseau's signs.
Options 2, 3, and 4 are not signs of
hypocalcemia.
A nurse is caring for a client following a 1. Indicates nerve damage
thyroidectomy. The client tells the nurse 2. Is harmless but permanent
that she is concerned because of voice 3. Will worsen before it subsides
hoarseness. The client asks the nurse *4. Is normal and will gradually subside*
whether the hoarseness will subside. *rationale* Hoarseness that develops in
The nurse appropriately tells the client the postoperative period is usually the
that the hoarseness: result of laryngeal pressure or edema

12 / 20
endocrine NCLEX Questions
Study online at https://quizlet.com/_96twcd
1. Indicates nerve damage and will resolve within a few days. The
2. Is harmless but permanent client should be reassured that the ef-
3. Will worsen before it subsides fects are transitory. Options 1, 2, and 3
4. Is normal and will gradually subside are incorrect.
1. Bradycardia
*2. Fever*
A nurse is monitoring a client with *3. Sweating*
Graves' disease for signs of thyrotoxic *4. Agitation*
crisis (thyroid storm). Which of the follow- 5. Pallor
ing signs and symptoms, if noted in the *rationale* Thyrotoxic crisis (thyroid
client, will alert the nurse to the presence storm) is an acute, potentially life-threat-
of this crisis? *Select all that apply.* ening state of extreme thyroid activi-
1. Bradycardia ty that represents a breakdown in the
2. Fever body's tolerance to a chronic excess of
3. Sweating thyroid hormones. The clinical manifes-
4. Agitation tations include fever greater than 100° F,
5. Pallor severe tachycardia, flushing and sweat-
ing, and marked agitation and restless-
ness. Delirium and coma can occur.
1. A client with hypothyroidism
*2. A client with Graves' disease who is
having surgery*
3. A client with diabetes mellitus sched-
Which of the following clients is at risk for uled for debridement of a foot ulcer
developing thyrotoxicosis? 4. A client with diabetes insipidus sched-
1. A client with hypothyroidism uled for an invasive diagnostic test
2. A client with Graves' disease who is *rationale* Thyrotoxicosis is usually seen
having surgery in clients with Graves' disease with the
3. A client with diabetes mellitus sched- symptoms precipitated by a major stres-
uled for debridement of a foot ulcer sor. This complication typically occurs
4. A client with diabetes insipidus sched- during periods of severe physiological
uled for an invasive diagnostic test or psychological stress such as trau-
ma, sepsis, the birth process, or major
surgery. It also must be recognized as
a potential complication following a thy-
roidectomy.
1. The reason for maintaining a diabetic
diet
13 / 20
endocrine NCLEX Questions
Study online at https://quizlet.com/_96twcd
*2. Instructions about early signs of a
wound infection*
3. Teaching regarding proper application
of an ostomy pouch
4. The need for lifelong replacement of all
A nurse is providing discharge instruc- adrenal hormones
tions to a client who had a unilateral *rationale* A client who is undergoing a
adrenalectomy. Which of the following unilateral adrenalectomy will be placed
will be a component of the instructions? on corticosteroids temporarily to avoid
1. The reason for maintaining a diabetic a cortisol deficiency. These medications
diet will be gradually weaned in the post-
2. Instructions about early signs of a operative period until they are discon-
wound infection tinued. Because of the anti-inflammato-
3. Teaching regarding proper application ry properties of corticosteroids, clients
of an ostomy pouch who undergo an adrenalectomy are at
4. The need for lifelong replacement of increased risk for developing wound in-
all adrenal hormones fections. Because of this increased risk
for infection, it is important for the client
to know measures to prevent infection,
early signs of infection, and what to do if
an infection is present. Options 1, 3, and
4 are incorrect instructions.
1. Inability to cope with the treatment
plan
2. Lack of sexual drive
3. Self-consciousness about body ap-
A nurse is caring for a client experienc-
pearance
ing thyroid storm. Which of the follow-
*4. Potential for cardiac disturbances*
ing would be a priority concern for this
*rationale* Clients in thyroid storm are
client?
experiencing a life-threatening event,
1. Inability to cope with the treatment
which is associated with uncontrolled
plan
hyperthyroidism. It is characterized by
2. Lack of sexual drive
high fever, severe tachycardia, delirium,
3. Self-consciousness about body ap-
dehydration, and extreme irritability. The
pearance
signs and symptoms of the disorder de-
4. Potential for cardiac disturbances
velop quickly, and therefore emergency
measures must be taken to prevent
death. These measures include main-
taining hemodynamic status and paten-
14 / 20
endocrine NCLEX Questions
Study online at https://quizlet.com/_96twcd
cy of airway as well as providing ade-
quate ventilation. Options 1, 2, and 3 are
not a priority in the care of the client in
thyroid storm.
1. Peripheral edema
2. Bilateral exophthalmos
A nurse is caring for a postoperative 3. Signs and symptoms of hypocalcemia
adrenalectomy client. Which of the fol- *4. Signs and symptoms of hypovolemia*
lowing does the nurse specifically moni- *rationale* Following adrenalectomy, the
tor for in this client? client is at risk for hypovolemia. Aldos-
1. Peripheral edema terone, secreted by the adrenal cortex,
2. Bilateral exophthalmos plays a major role in fluid volume balance
3. Signs and symptoms of hypocalcemia by retaining sodium and water. A defi-
4. Signs and symptoms of hypovolemia ciency of adrenocortical hormones does
not cause the clinical manifestations not-
ed in options 1, 2, and 3.
1. "Would you like me to ask the health
A client with myxedema has changes care provider for a prescription for a stim-
in intellectual function such as impaired ulant?"
memory, decreased attention span, and 2. "Give it time. I've seen dozens of
lethargy. The client's husband is upset clients with this problem that fully recov-
and shares his concerns with the nurse. er."
Which statement by the nurse is helpful 3. "I don't blame you for being frustrat-
to the client's husband? ed, because the symptoms will only get
1. "Would you like me to ask the health worse."
care provider for a prescription for a stim- *4. "It's obvious that you are concerned
ulant?" about your wife's condition, but the
2. "Give it time. I've seen dozens of symptoms may improve with continued
clients with this problem that fully recov- therapy."*
er." *rationale* Using therapeutic communi-
3. "I don't blame you for being frustrat- cation techniques, the nurse acknowl-
ed, because the symptoms will only get edges the husband's concerns and con-
worse." veys that the client's symptoms are com-
4. "It's obvious that you are concerned mon with myxedema. With thyroid hor-
about your wife's condition, but the mone therapy, these symptoms should
symptoms may improve with continued decrease, and cognitive function often
therapy." returns to normal. Option 1 is not help-
ful, and it blocks further communication.

15 / 20
endocrine NCLEX Questions
Study online at https://quizlet.com/_96twcd
Option 3 is pessimistic and untrue. Op-
tion 2 is not appropriate and offers false
reassurance.
*1. Help restore electrolyte balance.*
2. Make the body produce more cortisol.
A client with Addison's disease asks the 3. Replace insufficient circulating estro-
nurse how a newly prescribed medica- gens.
tion, fludrocortisone acetate (Florinef), 4. Alter the body's immune system func-
will improve the condition. When formu- tioning.
lating a response, the nurse should in- *rationale* Fludrocortisone acetate is a
corporate that a key action of this med- long-acting oral medication with min-
ication is to: eralocorticoid and moderate glucocor-
1. Help restore electrolyte balance. ticoid activity. It is prescribed for the
2. Make the body produce more cortisol. long-term management of Addison's dis-
3. Replace insufficient circulating estro- ease. Mineralocorticoids cause renal re-
gens. absorption of sodium and chloride ions
4. Alter the body's immune system func- and the excretion of potassium and hy-
tioning. drogen ions. These actions help restore
electrolyte balance in the body. The other
options are incorrect.
A client scheduled for a thyroidectomy 1. Inadequate knowledge about the sur-
says to the nurse, "I am so scared to get gical procedure
cut in my neck." Based on the client's *2. Fear about impending surgery*
statement, the nurse determines that the 3. Embarrassment about the changes in
client is experiencing which problem? personal appearance
1. Inadequate knowledge about the sur- 4. Lack of support related to the surgical
gical procedure procedure
2. Fear about impending surgery *rationale* The client is having a difficult
3. Embarrassment about the changes in time coping with the scheduled surgery.
personal appearance The client is able to express fears but is
4. Lack of support related to the surgical scared. No data in the question support
procedure options 1, 3, and 4.
1. A decrease in cortisol release
A nurse is caring for a client with Ad-
*2. A decreased secretion of aldos-
dison's disease. The nurse checks the
terone*
vital signs and determines that the client
3. An increase in epinephrine secretion
has orthostatic hypotension. The nurse
4. Increased levels of androgens
determines that this finding relates to
*rationale* A decreased secretion of al-
16 / 20
endocrine NCLEX Questions
Study online at https://quizlet.com/_96twcd
dosterone results in a limited reabsorp-
which of the following? tion of sodium and water; therefore the
1. A decrease in cortisol release client experiences fluid volume deficit. A
2. A decreased secretion of aldosterone decrease in cortisol, an increase in ep-
3. An increase in epinephrine secretion inephrine, and an increase in androgen
4. Increased levels of androgens secretion do not result in orthostatic hy-
potension.
1. Restricting fluid intake
2. Offering foods high in potassium
3. Checking family support systems
*4. Monitoring the blood glucose*
Which nursing measure would be ef- *rationale* The decrease in cortisol se-
fective in preventing complications in a cretion that characterizes Addison's dis-
client with Addison's disease? ease can result in hypoglycemia. There-
1. Restricting fluid intake fore monitoring the blood glucose would
2. Offering foods high in potassium detect the presence of hypoglycemia so
3. Checking family support systems that it can be treated early to prevent
4. Monitoring the blood glucose complications. Fluid intake should be en-
couraged to compensate for dehydra-
tion. Potassium intake should be restrict-
ed because of hyperkalemia. Option 3 is
not a priority for this client.
A nurse is collecting data on a client with *1. Depression*
a diagnosis of hypothyroidism. Which of 2. Nervousness
these behaviors, if present in the client's 3. Irritability
history, would the nurse determine as 4. Anxiety
being likely related to the manifestations *rationale* Hypothyroid clients experi-
of this disorder? ence a slow metabolic rate, and its mani-
1. Depression festation includes apathy, fatigue, sleepi-
2. Nervousness ness, and depression. Options 2, 3, and
3. Irritability 4 identify the clinical manifestations of
4. Anxiety hyperthyroidism.
1. Within normal limits
*2. A finding that needs to be reported
immediately*
3. An expected finding caused by the
operative stress response
4. Slightly abnormal but an insignificant
17 / 20
endocrine NCLEX Questions
Study online at https://quizlet.com/_96twcd
finding
*rationale* An adrenalectomy is per-
While collecting data on a client be- formed because of excess adrenal gland
ing prepared for an adrenalectomy, the function. Excess cortisol production im-
nurse obtains a temperature reading of pairs the immune response, which puts
100.8° F. The nurse analyzes this tem- the client at risk for infection. Because
perature reading as: of this, the client needs to be protect-
1. Within normal limits ed from infection, and minor variations
2. A finding that needs to be reported in normal vital sign values must be re-
immediately ported so that infections are detected
3. An expected finding caused by the early, before they become overwhelm-
operative stress response ing. In addition, the surgeon may elect to
4. Slightly abnormal but an insignificant postpone surgery in the event of a fever
finding because it can be indicative of infection.
Options 1, 3, and 4 are not correct inter-
pretations.
1. Stimulate the immune response.
*2. Promote electrolyte balance.*
3. Stimulate thyroid production.
4. Stimulate thyrotropin production.
*rationale* Florinef is a long-acting oral
medication with mineralocorticoid and
moderate glucocorticoid activity used for
The anticipated intended effect of fludro-
long-term management of Addison's dis-
cortisone acetate (Florinef) for the treat-
ease. Mineralocorticoids act on the re-
ment of Addison's disease is to:
nal distal tubules to enhance the reab-
1. Stimulate the immune response.
sorption of sodium and chloride ions and
2. Promote electrolyte balance.
the excretion of potassium and hydro-
3. Stimulate thyroid production.
gen ions. In small doses, fludrocortisone
4. Stimulate thyrotropin production.
acetate causes sodium retention and
increased urinary potassium excretion.
The client rapidly can develop hypoten-
sion and fluid and electrolyte imbalance
if the medication is discontinued abrupt-
ly. Options 1, 3, and 4 are not associated
with the effects of this medication.
1. Peanut butter, avocado, and red meat
*2. Skim milk, apples, whole-grain bread,

18 / 20
endocrine NCLEX Questions
Study online at https://quizlet.com/_96twcd
and cereal*
3. Organ meat, carrots, and skim milk
A nurse is caring for a client with hy- 4. Seafood, spinach, and cream cheese
pothyroidism who is overweight. Which *rationale* Clients with hypothyroidism
food items would the nurse suggest to may have a problem with being
include in the plan? over-weight because of their decreased
1. Peanut butter, avocado, and red meat metabolic need. They should consume
2. Skim milk, apples, whole-grain bread, foods from all food groups, which will
and cereal provide them with the necessary nutri-
3. Organ meat, carrots, and skim milk ents; however, the foods should be low
4. Seafood, spinach, and cream cheese in calories. Option 2 is the only option
that identifies food items that are low in
calories.
A client with Cushing's syndrome verbal- 1. "Don't be concerned, this problem can
izes concern to the nurse regarding the be covered with clothing."
appearance of the buffalo hump that has 2. "This is permanent, but looks are de-
developed. Which response by the nurse ceiving and not that important."
is appropriate? *3. "Usually, these physical changes
1. "Don't be concerned, this problem can slowly improve following treatment."*
be covered with clothing." 4. "Try not to worry about it. There are
2. "This is permanent, but looks are de- other things to be concerned about."
ceiving and not that important." *rationale* The client with Cushing's syn-
3. "Usually, these physical changes drome should be reassured that most
slowly improve following treatment." physical changes resolve with treatment.
4. "Try not to worry about it. There are Options 1, 2, and 4 are not therapeutic
other things to be concerned about." responses.
A client with newly diagnosed Cushing's
syndrome expresses concern about per- 1. This is a permanent feature.
sonal appearance, specifically about the 2. It can be minimized by wearing tight
"buffalo hump" that has developed at the clothing.
base of the neck. When counseling the *3. It may slowly improve with treatment
client about this manifestation, the nurse of the disorder.*
should incorporate the knowledge that: 4. It will quickly disappear once medica-
1. This is a permanent feature. tion therapy is started.
2. It can be minimized by wearing tight *rationale* The client with Cushing's syn-
clothing. drome should be reassured that most
3. It may slowly improve with treatment physical changes resolve over time with
of the disorder.

19 / 20
endocrine NCLEX Questions
Study online at https://quizlet.com/_96twcd
4. It will quickly disappear once medica- treatment. The other options are incor-
tion therapy is started. rect.
*1. Administering oxygen*
A client has just been admitted with a di-
2. Administering thyroid hormone
agnosis of myxedema coma. If all of the
3. Warming the client
following interventions were prescribed,
4. Giving fluid replacement
the nurse would place highest priority on
*rationale* As part of maintaining a
completing which of the following first?
patent airway, oxygen would be adminis-
1. Administering oxygen
tered first. This would be quickly followed
2. Administering thyroid hormone
by fluid replacement, keeping the client
3. Warming the client
warm, monitoring vital signs, and admin-
4. Giving fluid replacement
istering thyroid hormones.

20 / 20
Endocrine Disorder NCLEX Questions
Study online at https://quizlet.com/_6xhot

d. a serum somatomedin C (In-


A patient suspected of having
sulin-like-growth-factor) of more than
acromegaly has an elevated plasma
300
growth hormone level. In acromegaly,
(rationale- a normal response to growth
the nurse would also expect the patient's
hormone secretion is stimulation of
diagnostic results to include
the liver to produce somatomedin C
a. hyperinsulinemia
which stimulates growth of bones and
b. a plasma glucose of less than 70
soft tissue. The increased levels of so-
c. decreased growth hormone levels with
matomedin C normally inhibit growth
an oral glucose challenge test
hormone, but in acromegaly the pituitary
d. a serum sometomedin C (insulin-like
gland secretes GH despite elevated so-
growth-factor) of more than 300
matomedin C levels.)
c. undesirable changes in appearance
(Rationale- the increased production of
growth hormone in acromegaly causes
During assessment of the patient with an increase in thickness and width of
acromegaly, the nurse would expect the bones and enlargement of soft tissues,
patient to report resulting in marked changes in facial fea-
a. infertility tures, oily and coarse skin, and speech
b. dry, irritated skin difficulties. Height is not increased in
c. undesirable changes in appearance adults with growth hormone excess be-
d. an increase in height of 2 to 3 inches cause the epiphyses of the bones are
per year closed, and infertility is not a common
finding because growth hormone is usu-
ally the only pituitary hormone involved
in acromegaly.)
a. ensures that any clear nasal drainage
is tested for glucose
A patient with acromegaly is treated with
(Rationale- a transphenoidal hypophy-
a transphenoidal hypophysectomy. Post-
sectomy involves entry into the sella tur-
operatively, the nurse
cica through an incision in the upper lip
a. ensures that any clear nasal drainage
and gingiva into the floor of the nose
is tested for glucose
and the sphenoid sinuses. Postoperative
b. maintains the patient flat in bed to
clear nasal drainage with glucose con-
prevent cerebrospinal fluid leak
tent indicates CSF leakage from an open
c. assists the patient with toothbrushing
connection to the brain, putting the pa-
Q4H to keep the surgical area clean
tient at risk for meningitis. After surgery,
the patient is positioned with the head el-
1/9
Endocrine Disorder NCLEX Questions
Study online at https://quizlet.com/_6xhot
evated to avoid pressure on the sella tur-
cica, coughing and straining are avoid-
d. encourages deep breathing and
ed to prevent increased ICP and CSF
coughing to prevent respiratory compli-
leakage, and although mouth care is re-
cations
quired Q4H toothbrushing should not be
performed for 7-10post sx.)
a. monitor neurologic status Q2H or
more often if needed
During care of a patient with syn-
Rationale- the patient with SIADH
drome of inappropriate ADH (SIADH),
has marked dilution hyponatremia and
the nurse should
should be monitored for decreased neu-
a. monitor neurologic status Q2H or
rologic function and convulsions every 2
more often if needed
hours. ADH release is reduced by keep-
b. keep the head of the bed elevated to
ing the head of the bed flat to increase
prevent ADH release
left atrial filling pressure, and sodium
c. teach the patient receiving treatment
intake is supplemented because of hy-
with diuretics to restrict sodium intake
ponatremia and sodium loss caused by
d. notify the physician if the patient's
diuretics. A reduction in blood pressure
blood pressure decreases more than
indicates a reduction in total fluid volume
20mmHg from baseline
and is an expected outcome of treat-
ment.)
A patient with SIADH is treated with wa-
ter restriction and administration of IV
fluids. The nurses evaluates that treat-
ment has been effective when the pa- b. increased urine output, increased
tient experiences serum sodium, and decreased urine
a. increased urine output, decreased specific gravity
serum sodium, and increased urine spe- (rationale- the patient with SIADH has
cific gravity water retention with hyponatremia, de-
b. increased urine output, increased creased urine output and concentrated
serum sodium, and decreased urine urine with high specific gravity. improve-
specific gravity ment in the patient's condition reflected
c. decreased urine output, increased by increased urine output, normalization
serum sodium, and decreased urine of serum sodium, and more water in the
specific gravity urine, decreasing the specific gravity.)
d. decreased urine output, decreased
serum sodium, and increased urine spe-
cific gravity

2/9
Endocrine Disorder NCLEX Questions
Study online at https://quizlet.com/_6xhot
d. increase in urine osmolality
In a patient with central diabetes in- (rationale- a patient with DI has a de-
sipidus, administration of aqueous vaso- ficiency of ADH with excessive loss of
pressin during a water deprivation test water from the kidney, hypovolemia, hy-
will result in a pernatreamia, and dilute urine with a low
a. decrease in body weight specific gravity. When vasopressin is ad-
b. increase in urinary output ministered, the symptoms are reversed,
c. decrease in blood pressure with water retention, decreased urinary
d. increase in urine osmolality output that increases urine osmolality,
and an increase in blood pressure.)
c. a urine specific gravity of 1.002
A patient with DI is treated with nasal
(rationale- normal urine specific gravity
desmopression. The nurse recognize
is 1.003 to 1.030, and urine with a spe-
that the drug is not having an adequate
cific gravity of 1.002 is very dilute, indi-
therapeutic effect the the patient experi-
cating that there continues to be exces-
ences
sive loss of water and that treatment of
a. headache and weight gain
DI is inadequate. H/A, weight gain, and
b. nasal irritation and nausea
oral intake greater the urinary output are
c. a urine specific gravity of 1.002
signs of volume excess that occur with
d. an oral intake greater than urinary out-
overmedication. Nasal irritation & nau-
put
sea may also indicate overmedication.)
b. thiazide diuretics
(Rationale- in nephrogenic Di the kid-
When caring for a patient with nephro- ney is unable to respond to ADH, so
genic DI, the nurse would expect treat- vasopressin or hormone analogs are
ment to include not effective. Thiazide diuretics slow the
a. fluid restriction glomerular filtration rate in the kidney
b. thiazide diuretics and produce a decrease in urine out-
c. a high-sodium diet put. Low-sodium diets are also thought
d. chlorpropamide (DIabinese) to decrease urine output. Fluids are not
restricted, because the patient could be-
come easily dehydrated.)
A patient with Grave's dz asks the nurse d. "In genetically susceptible persons an-
what caused the disorder. The best re- tibodies form that attack thyroid tissue
sponse by the nurse is and stimulate overproduction of thyroid
a. "The cause of Grave's disease is not hormones."
known, although it is thought to be ge- (rationale- The antibodies present in
netic." Graves' disease that attack thyroid tissue
3/9
Endocrine Disorder NCLEX Questions
Study online at https://quizlet.com/_6xhot
b. "It is usually associated with goiter cause hyperplasia of the gland and stim-
formation from an iodine deficiency over ulate TSH receptors on the thyroid and
a long period of time." activate the production of thyroid hor-
c. "Antibodies develop against thyroid mones, creating hyperthyroidism. The
tissue and destroy it, causing a deficien- disease is not directly genetic, but indi-
cy of thyroid hormones" viduals appear to have a genetic sus-
d. "In genetically susceptible persons an- ceptibility to become sensitized to de-
tibodies form that attack thyroid tissue velop autoimmune antibodies. Goiter for-
and stimulate overproduction of thyroid mation from insufficient iodine intake is
hormones." usually associated with hypothyroidism.)
c. elevated temperature and signs of
heart failure
(rationale- a hyperthyroid crisis results
A patient is admitted to the hospital
in marked manifestations of hyperthy-
in thyrotoxic crisis. On physical assess-
roidism, with fever tachycardia, heart
ment of the patient, the nurse would ex-
failure, shock, hyperthermia, agitation,
pect to find
N/V/D, delirium, and coma. Although ex-
a. hoarseness and laryngeal stridor
ophthalmos may be present in the pa-
b. bulging eyeballs and arrhythmias
tient with Gravs' dz, it is not a significant
c. elevated temperature and signs of
factor in hyperthyroid crisis. Hoarsness
heart failure
and laryngeal stridor are characteristic
d. lethargy progressing suddenly to im-
of the tetany of hypoparathyroidism, and
pairment of consciousness
lethargy progressing to coma is charac-
teristic of myxedema coma, a complica-
tion of hypothyroidism.
a. how to support the head with the
hands when moving
Preoperative instructions for the patient
(rationale- to prevent strain on the su-
scheduled for a subtotal thyroidectomy
ture line postoperatively, the head must
includes teaching the patient
be manually supported while turning
a. how to support the head with the
and moving in bed, but range-of-mo-
hands when moving
tion exercise for the head and neck are
b. that coughing should due avoided to
also taught preoperatively to be grad-
prevent pressure on the incision
ually implemented after surgery. There
c. that the head and neck will need to
is no contraindication for coughing and
remain immobile until the incision heals
deep breathing, and they should be car-
d. that any tingling around the lips or in
rier out postoperatively. Tingling around
the lips or fingers is a sign of hypocal-

4/9
Endocrine Disorder NCLEX Questions
Study online at https://quizlet.com/_6xhot
cemia, which may occur if the parathy-
the fingers after surgery is expected and roid glands are inadvertently removed
temporary during surgery, and should be reported
immediately.)
c. avoid eating foods such as soybeans,
turnips, and rutabagas
(Rationale- when a patient has had a
When providing discharge instructions
subtotal thyroidectomy, thyroid replace-
to a patient following a subtotal thy-
ment therapy is not given, because ex-
roidectomy, the nurse advises the pa-
ogenous hormone inhibits pituitary pro-
tient to
duction of TSH and delays or prevents
a. never miss a daily dose of thyroid re-
the restoration of thyroid tissue regener-
placement therapy
ation. However, the patient should avoid
b. avoid regular exercise until thyroid
goitrogens, foods that inhibit thyroid,
function is normalized
such as soybeans, turnips, rutabagas,
c. avoid eating foods such as soybeans,
and peanut skins. REgular exercise stim-
turnips, and rutabagas
ulates the thyroid gland and is encour-
d. use warm salt water gargles several
age. Salt water gargles are used for
times a day to relieve throat pain
dryness and irritation of the mouth and
throat following radioactive iodine thera-
py.)
d. autoimmune-induced atrophy of the
gland
Causes of primary hypothyroidism in
(rationale- both Graves disease and
adults include
Hasimotos thyroiditis are autoimmune
a. malignant or benign thyroid nodules
disorders that eventually destroy the thy-
b. surgical removal or failure of the pitu-
roid gland, leading to primary hypothy-
itary gland
roidism. Thyroid tumors most often result
c. surgical removal or radiation of thyroid
in hyperthyroidism. Secondary hypothy-
gland
roidism occurs as a result of pituitary
d. autoimmune-induced atrophy of the
failure, and iatrogenic hypothyroidism re-
gland
sults from thyroidectomy or radiation of
the thyroid gland.)
d. decreased cardiac contractility and
Physical changes of hypothyroidism that coronary atherosclerosis
must be monitored when replacement (rationale- hypothyroidism affects the
therapy is started include heart in many ways, causing cardiomy-
opathy, coronary atherosclerosis, brady-
5/9
Endocrine Disorder NCLEX Questions
Study online at https://quizlet.com/_6xhot
cardia, pericardial effusions, and weak-
ened cardiac contractility. when thyroid
a. achlorhydria and constipation
replacement therapy is started, myocar-
b. slowed mental processes and lethargy
dial oxygen consumption is increased
c. anemia and increased capillary fragili-
and the resultant oxygen demand may
ty
cause angina, cardiac arrhythmias, and
d. decreased cardiac contractility and
heart failures. It is important to monitor
coronary atherosclerosis
patients with compromised cardiac sta-
tus when starting replacement therapy.)
b. provides written instruction for all infor-
A patient with hypothyroidism is treated mation related to the medication therapy
with Synthroid. When teaching the pa- (rationale- because of the mental slug-
tient about the therapy, the nurse gishness, inattentiveness, and memory
a. explains that caloric intake must be loss that occur with hypothyroidism, it
reduced when drug therapy is started is important to provide written instruc-
b. provides written instruction for all infor- tions and repeat information when teach-
mation related to the medication therapy ing the patient. Caloric intake can be in-
c. assures the patient that a return to nor- creased when drug therapy is started,
mal function will occur with replacement because of an increased metabolic rate,
therapy and replacement therapy must be taken
d. informs the patient that medications for life. Although most patients return to
must be taken until hormone balance is a normal state with treatment, cardio-
reestablished vascular conditions and psychoses may
persist.)
An appropriate nursing intervention for
the patient with hyperparathyroidism is
to b. increase fluid intake to 3000 to
a. pad side rails as a seizure precaution 4000ml/day
b. increase fluid intake to 3000 to (Rationale-A high fluid intake is indicated
4000ml/day in hyperparathyroidism to dilute hyper-
c. maintain bed rest to prevent patholog- calcemia and flush the kidneys so that
ic fractures calcium stone formation is reduced.)
d. monitor the patient for Trousseau's
phenomenon or Chvostek's sign
b. have the patient rebreathe in a paper
When the patient with parathyroid dis- bag
ease experiences symptoms of hypocal- (rationale- rebreathing in a paper bag
promotes carbon dioxide retention in the
6/9
Endocrine Disorder NCLEX Questions
Study online at https://quizlet.com/_6xhot
blood, which lowers pH and creates an
cemia, a measure that can be used to acidosis. An academia enhances the sol-
temporarily raise serum calcium levels is ubility and ionization of calcium, increas-
to ing the proportion of total body calcium
a. administer IV normal saline available in physiologically active form
b. have the patient rebreathe in a paper and relieving the symptoms of hypocal-
bag cemia. Saline promotes calcium excre-
c. administer Lasix as ordered tion, as does Lasix. Phosphate levels in
d. administer oral phosphorous supple- the blood are reciprocal to calcium and
ments an increase in phosphate promotes cal-
cium excretion.)
A patient is admitted to the hospital with a. HTN, peripheral edema, and petechi-
a diagnosis of Cushing syndrome. On ae
physical assessment of the patient, the (rationale- The effects of glucocorticoid
nurse would expect to find excess include weight gain from accu-
a. HTN, peripheral edema, and petechi- mulation and redistribution of adipose
ae tissue, sodium and water retention, glu-
b. weight loss, buffalo hump, and moon cose intolerance, protein wasting, loss
face with acne of bone structure, loss of collagen, and
c. abdominal and buttock striae, truncal capillary fragility. Clinical manifestations
obesity, and hypotension of corticosteroid deficiency include hy-
d. anorexia, signs of dehydration, and potension, dehydration, weight loss, and
hyper pigmentation of the skin hyperpigmentation of the skin.)
c. cardiac arrhythmias
(rationale- electrolyte changes that oc-
To prevent complications in the patient
cur in Cushing syndrome include sodi-
with Cushing syndrome, the nurse mon-
um retention and potassium excretion
itors the patient for
by the kidney, resulting in hypokalemia,
a. hypotension
which may lead to cardiac arrhythmias or
b. hypoglycemia
arrest. Hypotension, hypoglycemia, and
c. cardiac arrhythmias
decreased cardiac strength and output
d. decreased cardiac output
are characteristic of adrenal insufficien-
cy.)
A patient is scheduled for bilateral c. increased to promote an adequate re-
adrenalectomy. During the postoperative sponse to the stress of surgery
period, the nurse would expect adminis- (rationale- although the patient with
tration of corticosteroids to be Cushing syndrome has excess corticos-
a. reduced to promote wound healing teroids, removal of the glands and the
7/9
Endocrine Disorder NCLEX Questions
Study online at https://quizlet.com/_6xhot
stress of surgery require that high doses
b. withheld until symptoms of hypocorti-
of cortisone be administered postopera-
solism appear
tively for several days. The nurse should
c. increased to promote an adequate re-
monitor the patient postoperatively to de-
sponse to the stress of surgery
tect whether large amounts of hormones
d. reduced because excessive hor-
were released during surgical manipula-
mones are released during surgical ma-
tion and to ensure the healing is satisfac-
nipulation of the glands
tory.)
c. IV administration of hydrocortisone
(rationale- vomiting and diarrhea are
early indicators of addisonian crisis and
A patient with Addison's disease comes
fever indicates an infection, which s
to the emergency department with com-
causing additional stress for the patient.
plaints of N/V/D, and fever. The nurse
treatment of a crisis requires immediate
would expect collaborative care to in-
glucocorticoid replacement, and IV hy-
clude
drocortisone, fluids, sodium and glucose
a. parenteral injections of ACTH
are necessary for 24hours. Addison's
b. IV administration of vasopressors
disease is a primary insufficiency of the
c. IV administration of hydrocortisone
adrenal gland, and ACTH is not effective,
d. IV administration of D5W with 20mEq
nor would vasopressors be effective with
of KCl
the fluid deficiency of Addison's. Potassi-
um levels are increased in Addison's dz,
and KCl would be contraindicated.)
a. the patient appears alert and oriented
(rationale- confusion, irritability, disorien-
The nurse determines that the patient in
tation, or depressioni s often present
acute adrenal insufficiency is respond-
in the patient with Addison's dz, and a
ing favorably to treatment when
positive response to therapy would be
a. the patient appears alert and oriented
indicated by a return to alertness and
b. the patient's urinary output has in-
orientation. Other indication of response
creased
to therapy would be a decreased urinary
c. pulmonary edema is reduced as evi-
output, decreased serum potassium,
denced by clear lung sounds
and increased serum sodium and glu-
d. laboratory tests reveal serum eleva-
cose. The patient with Addison's would
tions of K and glucose and a decrease
be very dehydrated and volume-deplet-
in sodium
ed and would not have pulmonary ede-
ma.)

8/9
Endocrine Disorder NCLEX Questions
Study online at https://quizlet.com/_6xhot
b. monitoring blood pressure38
The most important nursing intervention
(rationale- a pheochromocytoma is a
during the medical and surgical treat-
catecholamine-producing tumor of the
ment of the patient with a pheochromo-
adrenal medulla, which may cause se-
cytoma is
vere, episodic HTN; severe, pounding
a. administering IV fluids
headache; and profuse sweating. Mon-
b. monitoring blood pressure
itoring for dangerously high BP before
c. monitoring I&O and daily weights
surgery is critical, as is monitoring for BP
d. administering B-adrenergic blocking
fluctuation during medical and surgical
agents
tx.)
a. Lasix37
(rationale- hyperaldosteronism is an ex-
cess of aldosterone, which is man-
When caring for a patient with prima-
ifested by sodium and water reten-
ry hyperaldosteronism, the nurse would
tion and potassium excretion. Lasix
question a physician's order for the use
is a potassium-wasting diuretic that
of
would increase the potassium deficiency.
a. Lasix
Aminoglutethimide blocks aldosterone
b. amiloride (midamor)
synthesis; amiloride is apotassium-spar-
c. spironolactone (aldactone)
ing diuretic; and spironolactone blocks
d. aminoglutethimide (cytadren)
mineralocorticoid receptors in the kid-
ney, increasing secretion of sodium and
water and retention of potassium.)

9/9
Endocrine NCLEX Questions
Study online at https://quizlet.com/_3kg1kg

1. D. "Ketone test- The diabetic patient has been instructed on ketone testing.
ing should be Which statement indicates the need for further instruc-
done each morn- tion?
ing when I am in A. "Ketones indicate periods of poor glycemic control."
a fasting state." B. "I should perform ketone testing whenever I am hyper-
glycemic."
C. "Performing ketone testing during periods of illness is
recommended."
D. "Ketone testing should be done each morning when I
am in a fasting state."

2. C. Good The hemoglobin A1c test results of the patient are 8.2%.
The nurse knows which word best describes the patient's
glycemic control?
A. Fair
B. Poor
C. Good
D. Excellent

3. A. 2-hour test A patient has completed glucose tolerance testing. The


results of 175 results are normal. Which result is reflective of this finding?
mg/dL A. 2-hour test results of 175 mg/dL
B. 2-hour test results of less than 225 mg/dL
C. Fasting blood glucose of 50 mg/dL
D. Fasting blood glucose of 130 mg/dL

4. C. "Fasting is re- A patient is scheduled to have a glucose tolerance test.


quired for 10 to What information should be included in the instructions
12 hours before provided to the patient?
the test." A. "Eat a normal-sized breakfast before the test."
B. "Limit carbohydrate intake 5 to 7 days before the test."
C. "Fasting is required for 10 to 12 hours before the test."
D. "During the test, you may eat low-carbohydrate
snacks."

5. C. To evaluate the A patient is admitted to the hospital with a suspected di-


thyroid's ability agnosis of hyperthyroidism. She asks the LPN/LVN about
to accumulate io- the thyroid scan for which she is scheduled that morning.
dine The nurse should tell her that it is done for which purpose?
1 / 11
Endocrine NCLEX Questions
Study online at https://quizlet.com/_3kg1kg
A. To collect a blood sample for analysis
B. To determine underactivity of the thyroid cells
C. To evaluate the thyroid's ability to accumulate iodine
D. To detect abnormal areas of growth or tissue in the
thyroid

6. C. Decreased The nurse who works primarily with older adults knows
ability to respond that which of the following is a significant age-related
to stress change of the endocrine system?
A. Change in glucose production
B. Inconsistent hormone production
C. Decreased ability to respond to stress
D. Increased number of endocrine disorders

7. D. Epinephrine When an individual is under a great deal of stress for a pro-


longed period of time, which hormone would be expected
to be secreted in increased amounts?
A. Insulin
B. Estrogen
C. Thyroxine
D. Epinephrine

8. D. Parathyroid When a patient has alterations in serum calcium and


phosphorus levels, which gland is dysfunctional?
A. Thyroid
B. Adrenal
C. Pancreas
D. Parathyroid

9. B. Asking the A patient is scheduled for thyroid function tests in the


patient if she morning. Patient preparation for this diagnostic procedure
has recently eat- includes which interventions? (Select all that apply.)
en shrimp or A. Ascertaining if the patient is claustrophobic
clams Instructing the patient to discard her first morning urine
D. Explaining to B. Asking the patient if she has recently eaten shrimp or
the patient that clams
the dose of ra- C. Keeping the patient on NPO status (nothing by mouth)
dioactivity she after midnight
will be exposed D. Explaining to the patient that the dose of radioactivity
she will be exposed to during testing is very small
2 / 11
Endocrine NCLEX Questions
Study online at https://quizlet.com/_3kg1kg
to during testing
is very small

10. A. Cortisol can The nurse is caring for a patient with a deficiency of
be classified as a cortisol. The nurse correctly recognizes which information
glucocorticoid. about this disorder? (Select all that apply.)
B. This patient
is at an increased A. Cortisol can be classified as a glucocorticoid.
risk for hyper- B. This patient is at an increased risk for hyperglycemia.
glycemia. C. Cortisol levels are controlled by the posterior pituitary
E. Cortisol works gland.
to counteract the D. The patient has an increased chance of becoming
body's response overweight.
to inflammation E. Cortisol works to counteract the body's response to
inflammation

11. A. Keeping en- A patient who has hyperthyroidism exhibits symptoms of


vironmental stim- anxiety, nervousness, and agitation. Which intervention
uli to a minimum should be included in the patient's care?
A. Keeping environmental stimuli to a minimum
B. Encouraging questions about options for treatment
C. Stressing the importance of complying with the treat-
ment regimen
D. Maintaining the temperature of the room slightly above
normal

12. C. Vasopressin A patient has diabetes insipidus (DI). The LPN/LVN should
plan to provide information on which medication?
A. Insulin
B. Cortisone
C. Vasopressin
D. Fludrocortisone acetate

13. D. Insufficiencies A patient is diagnosed as having Addison disease. This


of the mineralo- condition is the result of which disturbance?
corticoids and A. A secreting tumor of the adrenal cortex
glucocorticoids B. Overfunctioning of the pituitary and hypothalamus
C. Excessive secretion of adrenocorticotropic hormone
D. Insufficiencies of the mineralocorticoids and glucocor-
ticoids
3 / 11
Endocrine NCLEX Questions
Study online at https://quizlet.com/_3kg1kg

14. D. Extreme tem- In the postoperative period, the LPN/LVN should observe
perature eleva- a patient who has had a thyroidectomy for which signs of
tion and rapid thyroid crisis?
pulse rate A. Depression and fatigue
B. Respiratory distress and hoarseness
C. Twitching of muscles and severe convulsions
D. Extreme temperature elevation and rapid pulse rate

15. C. Benign pitu- The patient presents to the clinic with acromegaly, mus-
itary adenoma cle weakness, and osteoporosis. Laboratory results show
elevated growth hormone levels. These symptoms, along
with computed tomography scan findings, most closely
correlate with which disorder?
A. Astrocytoma
B. Craniopharyngioma
C. Benign pituitary adenoma
D. Malignant pituitary adenoma

16. A. Semi-Fowler A patient has just returned from the postanesthesia care
unit after a hypophysectomy. The nurse knows to keep the
patient in which position?
A. Semi-Fowler
B. Left lateral Sims
C. Supine with no pillows
D. Reverse Trendelenburg

17. B. Providing the The nurse is supervising the care of a patient by unli-
patient with tis- censed assistive personnel (UAP) after hypophysectomy.
sues to blow her Which action by the UAP would prompt the nurse to inter-
nose vene immediately?
A. Encouraging the patient to drink water regularly
B. Providing the patient with tissues to blow her nose
C. Assisting the patient to the toilet while remaining up-
right
D. Reminding the patient to wear sequential compression
devices while in bed

18. A. Iodine The patient presents to the clinic with a gross enlargement
of the anterior neck. The nurse knows that this is most
4 / 11
Endocrine NCLEX Questions
Study online at https://quizlet.com/_3kg1kg
likely caused by a deficiency in which substance?
A. Iodine
B. Sodium
C. Calcium
D. Vitamin D

19. A. Increased The nurse is reviewing the history of a patient suspected


heart rate of having hyperthyroidism. Which manifestation(s) would
B. Increased ap- be supportive of the diagnosis? (Select all that apply.)
petite A. Increased heart rate
C. Emotional in- B. Increased appetite
stability C. Emotional instability
E. Hyperactivi- D. Mental sluggishness
ty with increas- E. Hyperactivity with increasing sense of fatigue
ing sense of fa-
tigue

20. A. Urine osmolal- The patient presents to the emergency department with
ity symptoms of syndrome of inappropriate antidiuretic hor-
B. Serum osmo- mone (SIADH). The nurse knows to educate the patient
lality about which test(s) or procedure(s) that will be performed
before a diagnosis can be made? (Select all that apply.)
A. Urine osmolality
B. Serum osmolality
C. Glucose tolerance testing
D. Magnetic resonance imaging (MRI)
E. Radioactive iodine uptake test (RAIU)

21. B. Regular A patient who is undergoing surgery will have an intra-


venous solution to which insulin will be added. Which type
of insulin must be used?
A. Lente
B. Regular
C. Ultralente
D. Neutral protamine Hagedorn (NPH)

22. D. Thirst, dry When teaching a patient with type 1 diabetes about home
mucous mem- care, the LPN/LVN would be sure to include which signs
branes, and dry of diabetic ketoacidosis?
skin A. Dark, scanty urine, and diarrhea
5 / 11
Endocrine NCLEX Questions
Study online at https://quizlet.com/_3kg1kg
B. Cool, clammy skin, and nervousness
C. Hunger, headache, and tremulousness
D. Thirst, dry mucous membranes, and dry skin

23. C. She can play A patient with type 1 diabetes mellitus (DM) plays tennis
tennis, but she and asks if she will be able to continue with that sport. The
will need to eat nurse should base his response on which information?
more before she A. It would be better to take up walking or some quiet
plays sport.
B. She can play tennis, but she will need an extra dose of
insulin.
C. She can play tennis, but she will need to eat more
before she plays.
D. She cannot play tennis because heavy exercise is not
permitted with this type of diabetes.

24. D. It can often be In discussing DM with a patient, it is important to base the


controlled by diet discussion on which information regarding the disease?
and regular exer-
cise. A. It can often be cured by insulin therapy.
B. It has no cure and is considered "hopeless."
C. It has no specific treatment other than use of insulin.
D. It can often be controlled by diet and regular exercise.

25. C. "Hypo- A patient recently diagnosed as having hypoglycemia


glycemia has says, "Hypoglycemia! I can't live with that. My neighbor,
been successful- Joseph, had that and he acted crazy!" Which response by
ly treated by diet the nurse is most appropriate?
modifications
A. "You seem to be overreacting to the problem."
B. "You're right; it would be difficult to live with hypo-
glycemia."
C. "Hypoglycemia has been successfully treated by diet
modifications."
D. "Taking care of yourself years ago would have prevent-
ed the problem"

26. B. "I will draw A patient newly diagnosed with diabetes is learning to
up the regular in- administer his injections of NPH and regular insulin.
Which statement indicates that the patient understands
6 / 11
Endocrine NCLEX Questions
Study online at https://quizlet.com/_3kg1kg
sulin before the the nurse's teaching regarding proper insulin administra-
NPH." tion?
A. "I will draw up the NPH before the regular insulin."
B. "I will draw up the regular insulin before the NPH."
C. "I will give myself the NPH and the regular insulin in two
different injections."
D. "It doesn't matter which insulin I draw up first, as long
as the amount is correct.

27. A. Diabetic neu- A patient with diabetes is admitted to the emergency de-
ropathy partment with complaints of lack of feeling, yet debilitating
pain in his legs and feet, constipation, and sexual impo-
tence. These symptoms most closely correlate with which
disorder?

A. Diabetic neuropathy
B. Diabetic retinopathy
C. Diabetic ketoacidosis
D. Diabetic nephropathy

28. C. "Inspect each A patient with diabetes asks her nurse about foot care
foot daily for when she is discharged home. What is the nurse's best
cuts, cracks, blis- response?
ters, or abra-
sions." A. "Cut your toenails in a V shape to prevent ingrown
toenails."
B. "Soak your feet in hot water each night before going to
bed."
C. "Inspect each foot daily for cuts, cracks, blisters, or
abrasions."
D. "There are no special instructions for your feet when
you have diabetes"

29. C. Excessive The nurse is teaching a class on diabetes to a group of


thirst adults in the community. The nurse should be sure to in-
D. Frequent uri- clude information on which classic symptoms of diabetes?
nation (Select all that apply.)
E. Increased ap- A. Hypertension
petite B. Vision changes

7 / 11
Endocrine NCLEX Questions
Study online at https://quizlet.com/_3kg1kg
C. Excessive thirst
D. Frequent urination
E. Increased appetite
F. Recurrent urinary tract infections (UTIs)

30. C. Recurrent in- The nurse is providing discharge teaching to a patient


fections recently diagnosed with type 2 DM. The nurse should
D. Delayed include information on which long-term consequences of
wound healing poor glycemic control? (Select all that apply.)
E. Peripheral A. Depression
vascular disease B. Hypertension
C. Recurrent infections
D. Delayed wound healing
E. Peripheral vascular disease

31. C. Treat hypocal- The nurse is caring for a client after a thyroidectomy and
cemic tetany notes that calcium gluconate is prescribed for the client.
The nurse determines that this medication has been pre-
scribed for which reason?
A. Treat thyroid storm
B. prevent cardiac irritability
C. Treat hypocalcemic tetany
D. Stimulate the release of parathyroid hormone

32. D. Reassure the The nurse is collecting data regarding a client after a
client that is is thyroidectomy and notes that the client has developed
usually tempo- hoarseness and a weak voice, Which nursing action is
rary condition appropriate?
A. Check for signs of bleeding
B. Administer calcium gluconate
C. Notify the registered nurse immediately
D. Reassure the client that is is usually temporary condi-
tion

33. B. Maintain a A client is admitted to the ER and a diagnosis of myxede-


patent airway ma coma is made. Which action should the nurse prepare
to carry out initially?
A. Warm the client
B. Maintain a patent airway

8 / 11
Endocrine NCLEX Questions
Study online at https://quizlet.com/_3kg1kg
C. Monitor Intravenous fluids
D. Administer thyroid hormone

34. C. Apply a mois- The nurse is assisting with preparing a teaching plan for
turizing lotion to the client with diabetes mellitus regarding proper foot care.
dry feet, but not Which instruction should include in the plan of care?
in between the A. Soak the feet in hot water
toes B. Avoid using soap to wash the feet
C. Apply a moisturizing lotion to dry feet, but not in be-
tween the toes
D. Always have a podiatrist cut your toesnails; never cut
them yourself

35. C. " I need to The nurse provides dietary instructions to a client with
buy special dia- diabetes mellitus regarding the prescribed diabetic diet.
betic foods" Which statement made by the client indicated the need for
further teaching?
A. " Ill eat a balanced meal plan"
B. " I need to drink diet soft drinks"
C. " I need to buy special diabetic foods"
D. " I will snack on fruit instead of cake"

36. B. Rotate the A client who has been newly diagnosed with diabetes
insulin injection mellitus has been stabilized with daily insulin injections.
sites systemati- Which teaching information should the nurse reinforce
cally upon discharge?
A. Keep insulin vials refrigerated at all times
B. Rotate the insulin injection sites systematically
C. Increase the amount of insulin before unusual exercise
D. Monitor urine acetone level to determine the insulin
dosage

37. B. Shakiness The nurse reinforces teaching with a client with diabetes
mellitus regarding differentiating between hypoglycemia
and ketoacidosis. The client demonstrates an understand-
ing of the teaching by stating that glucose will be taken if
which symptom develops?
A. Polyuria
B. Shakiness

9 / 11
Endocrine NCLEX Questions
Study online at https://quizlet.com/_3kg1kg
C. Blurred Vision
D. Fruity breath odor

38. D. I will no- When the nurse is reinforcing instructions to a client who
tify my health has been newly diagnosed with type 1 diabetes melli-
care provider if tus, which statement by the client would indicate that the
my blood glu- teaching has been effective?
cose level is con- A. " I will stop taking my insulin if im too sick to eat."
sistently greater B. " I will decrease my insulin during times of illness."
than 250mg/dL C. " I will adjust my insulin dose according to the level of
glucose in my urine."
D. I will notify my health care provider if my blood glucose
level is consistently greater than 250mg/dL

39. A. " I am urinating The nurse is monitoring a client who has been newly
a lot" diagnosed with diabetes mellitus for signs of complica-
tions. Which statement made by the client would indicate
hyperglycemia and thus warrant health care provider no-
tification?
A. " I am urinating a lot"
B. " My pulse is really slow"
C. " I am sweating for no reason"
D. " My blood pressure is really high"

40. C. Monitor blood The nurse is reinforcing instructions with a client with
glucose levels diabetes mellitus who is recovering from DKA regarding
frequently measures to prevent a recurrence. Which instruction is
important for the nurse to emphasize?
A.Eat six small meals daily
B. Test the urine ketones level
C. Monitor blood glucose levels frequently
D. Receive appropriate follow- up health care

41. A. " I can eat The nurse is reinforcing discharge teaching with a client
foods that con- who has Cushings Syndrome. Which statement by the
tain potassium." client indicated that the instructions related to dietary man-
agement were understood?
A. " I can eat foods that contain potassium."
B. " I will need to limit the amount of protein in my diet."
C. " I am fortunate that i can eat all the salty foots I enjoy."
10 / 11
Endocrine NCLEX Questions
Study online at https://quizlet.com/_3kg1kg
D. " I am fortunate that I do not need to follow my special
diet."

42. A. Dry Skin The nurse educator is asking the nursing student to recall
E. Constipated the signs and symptoms of hypothyroidism. The nurse
F. Cold intoler- educator determines that the student understands this
ance disorder if which are included in the students response?
select all that apply
A. Dry skin
B. Irritability
C. Palpation
D. Weight loss
E. Constipated
F. Cold intolerance

43. B. Laryngeal stri- The nurse is caring for a post-op parathyroidectomy client.
dor Which would require the nurses immediate attention?
A. Incision pain
B. Laryngeal stridor
C. Difficulty voiding
D. Abdominal cramps

44. A. Plan for injec- The nurse notes that a client with type 1 diabetes mellitus
tion rotation has lipodystrophy on both upper thighs. Which further
information should the nurse obtain from the client during
data collection?
A. Plan for injection rotation
B. Consistency of aspiration
C. Preparation of injection site
D. Angle at which the medication is administered

45. D." I should not A client with type 1 diabetes mellitus calls the nurse to re-
exercise in the port recurrent episodes of hypoglycemia. Which statement
late afternoon" by the client indicated a correct understanding of Humulin
N insulin and exercise?
A. " I should not exercise after lunch"
B. " I should not exercise after breakfast."
C." I should not exercise in the late evening"
D." I should not exercise in the late afternoon"

11 / 11
Endocrine NCLEX Questions
Study online at https://quizlet.com/_1u2oh0

1. The patient has a hormone deficiency. Which deficien- C. Thyroid-stimu-


cy is highest priority? lating hormone
A. Growth hormone
B. Luteinizing hormone
C. Thyroid-stimulating hormone
D. Follicle-stimulating hormone

2. Which of the following would be an expected thera- C. Urine output de-


peutic response of the patient being treated for dia- creased; specific
betes insipidus with antidiuretic hormone? gravity increased
A. Urine output is increased; specific gravity is in-
creased
B. Urine output is increased; specific gravity is de-
creased
C. Urine output is decreased; specific gravity is in-
creased
D. Urine output is increased; specific gravity is de-
creased

3. Which serum electrolyte values alert the nurse to the A. Serum sodi-
possibility of hypoaldosteronism? um 150 mmol/L;
A. Serum sodium 150 mmol/L, serum potassium 2.5 serum potassium
mmol/L 2.5 mmol/L
B. Serum sodium 140 mmol/L, serum potassium 5.0
mmol/L
C. Serum sodium 130 mmol/L, serum potassium 2.5
mmol/L
D. Serum sodium 130 mmol/L, serum potassium 7.5
mmol/L

4. Which alteration in vital signs is the most indicative of D. Apical heart


hypothyroidism rate of 50 beats/
A. Temperature of 99.9ÚF minute
B. Respiratory rate of 26 breaths/min
C. Blood pressure of 148/90 mmHg
D. Apical heart rate of 50 beats/min

5. The nurse asseses for which clinical manifestation


in a patient with uncontrolled diabetes melitus and
1/5
Endocrine NCLEX Questions
Study online at https://quizlet.com/_1u2oh0
ketoacidosis C. Increased rate
A. Severe orthostatic hypotension and depth of respi-
B. Oral temperature of 102ÚF rations
C. Increased rate and depth of respirations
D. Extremity tremors followed by seizure activity

6. The nurse determines that which arterial blood gas B. pH 7.28, HCOƒ
values are consistent with ketoacidosis in the patient 18 mEq/L, PCO‚ 38
with diabetes? mmHg
A. pH 7.38, HCOƒ 22 mEq/L, PCO‚ 38 mmHg
B. pH 7.28, HCOƒ 18 mEq/L, PCO‚ 38 mmHg
C. pH 7.48, HCOƒ 28 mEq/L, PCO‚ 38 mmHg
D. pH 7.28, HCOƒ 22 mEq/L, PCO‚ 58 mmHg

7. Which dietary modification will the nurse provide for C. Increased calo-
the patient with hyperthyroidism? ries, proteins and
A. Decreased calories and proteins and increased car- carbohydrates
bohydrates
B. Elimination of carbohydrates and increased pro-
teins and fats
C. Increased calories, proteins and carbohydrates
D. Supplemental vitamins and reduction of calories

8. The nurse monitors for which laboratory disorder in D. Serum potas-


the patient receiving IV insulin for hyperglycemia? sium level of 2.5
A. Serum chloride level of 90 mmol/L mmol/L
B. Serum calcium level of 8.0 mg/dL
C. Serum sodium level of 135 mmol/L
D. Serum potassium level of 2.5 mmol/L

9. Which condition may cause hypopituitarism? (select D. Autoimmune


all that apply) conditions
A. Tumors E. Tuberculosis
B. Diplopia
C. Infertility
D. Autoimmune conditions
E. Tuberculosis

10. Which of the following does not occur in Addison's A. Exophthalmos


disease?
2/5
Endocrine NCLEX Questions
Study online at https://quizlet.com/_1u2oh0
A. Exophthalmos
B. Increased skin pigmentation
C. Low levels of serum ACTH
D. Low levels of serum cortisol

11. Signs and symptoms of diabetes mellitus includes all D. Swollen ankles
except
A. Frequent urination
B. Increased glucose
C. Increased thirst
D. Swollen ankles

12. Which test provides a good measure of blood glucose C. Glycosylated


levels over the previous 2 to 3 months? hemoglobin test
A. Glucose tolerance test (GTT) (HgbA1C)
B. Fasting blood glucose test
C. Glycosylated hemoglobin test (HgbA1C)
D. Capillary blood monitoring test

13. What is the primary cause of type 1 diabetes? D. Destruction of


A. Chronic obesity pancreatic en-
B. Increased glucose production in the liver zymes by an au-
C. Increased resistance of body cells to insulin action toimmune reaction
D. Destruction of pancreatic enzymes by an autoim-
mune reaction

14. Which of the following conditions may cause immuno- D. Cushing's syn-
suppression? drome
A. Acromegaly
B. Hyperthyroidism
C. Diabetes insipidus
D. Cushing's syndrome

15. Your patient has acromegaly. You would expect A. Your patient to
A. Your patient to have an overactive pituitary gland have an overactive
B. Your patient to have an under active pituitary gland pituitary gland
C. Your patient to have an overactive adrenal gland
D. Your patient to have an under active adrenal gland

16. B. Hypertension
3/5
Endocrine NCLEX Questions
Study online at https://quizlet.com/_1u2oh0
A patient with pheochromocytoma is scheduled to
have the tumor surgically removed. During the admis-
sion assessment, which finding would the nurse most
likely observe?
A. Hyperkalemia
B. Hypertension
C. Hyperthermia
D. Bradycardia

17. The nurse assesses the patient with a diagnosis of C. Fever, tachycar-
thyroid storm. Which classic signs and symptoms as- dia, and abdomi-
sociated with thyroid storm indicate the priority need nal pain
for immediate nursing intervention?
A. Polyuria, nausea and severe headache
B. Hypotension, translucent skin and obesity
C. Fever, tachycardia, and abdominal pain
D. Profuse diaphoresis, flushing and constipation

18. The nurse is aware that the following is the most D. An adrenal tu-
common cause of hyperaldosteronism. mor
A. Deficient potassium intake
B. Excessive sodium intake
C. A pituitary tumor
D. An adrenal tumor

19. An incoherent female with a history of hypothyroidism C. Myxedema


is brought to the emergency department by the res- coma
cue squad. Physical and laboratory findings reveal
hypothermia, hypoventilation, respiratory acidosis,
bradycardia, and hypotension. Knowing that these
findings suggest sever hypothyroidism, the nurse pre-
pares to take emergency action to prevent the poten-
tial complication of
A. Hashimoto's thyroiditis
B. Cushing's syndrome
C. Myxedema coma
D. Thyroid storm

20. You are taking care of a patient who presents to


the emergency department with suspected hyperthy-
4/5
Endocrine NCLEX Questions
Study online at https://quizlet.com/_1u2oh0
roidism. You expect the laboratory values for this pa- B. Decreased
tient to be TSH, increased Tƒ
A. Increased TSH, increased Tƒ and T„ and T„
B. Decreased TSH, increased Tƒ and T„
C. Increased TSH, decreased Tƒ and T„
D. Decreased TSH, decreased Tƒ and T„

5/5
Endocrine NCLEX Questions
Study online at https://quizlet.com/_221wbp

1. The nurse caring for a patient who recently underwent Answer: B


removal of a pituitary adenoma via the transphenoidal
approach knows that which of the following routine
post-operative interventions will be contraindicated
for this patient:

A.Turn every 2 hours


B.Cough and deep breath
C.Ambulation
D.HOB 30 degrees

2. The nurse caring for a patient who recently underwent Answer. D


removal of a pituitary adenoma via the transphenoidal
approach knows that a common complication with
this surgery is a headache. In order to prevent this
complication the nurse will:

A.Provide pain medication routinely


B.Keep the patient in the supine position
C.Assess VS every 2 hours
D.Keep the patient's HOB at 30 degrees

3. The nurse caring for a patient admitted with an ADH D


secreting lung cancer would anticipate which lab find-
ing:

A. Serum sodium 150


B. Serum osmolality elevated
C. Urine specific gravity 1.002
D. Serum sodium 125

4. Common nonspecific manifestations that may alert C


the nurse to endocrine dysfunction include:

A. Goiter and alopecia


B. Exophthalmos and tremors
C. Weight loss, fatigue, depression
D. Polyuria, polydipsia, and polyphagia

1/5
Endocrine NCLEX Questions
Study online at https://quizlet.com/_221wbp
5. A patient with diabetes insipidus is treated with Answer: C
DDAVP. The nurse determines that the drug is not hav-
ing an adequate therapeutic effect when the patient
experiences:

A. Headache and weight gain


B. Nasal irritation and nausea
C. A urine specific gravity of 1.002 (1.003-1.030)
D. Oral intake greater than urinary output

6. The nurse caring for a patient admitted with SIADH Answer: C


can anticipate which of the following physician or-
ders:

A.0.45% NS at 100 ml/hr


B.D5W at 100 ml/hr
C.Fluid restriction of 1000 ml/day
D.DDAVP IVP

7. A nurse is caring for a patient after hypophysectomy. Answer: B


The nurse notices clear nasal drainage from the pa-
tient's nostril. The initial nursing action would be to:

A. Lower the head of the bed


B. Test the drainage for glucose
C. Obtain a culture of the drainage
D. Continue to observe the drainage

8. When obtaining subjective data from a patient during Answer: A


assessment of the endocrine system, the nurse asks
specifically about:

A. Energy level
B. Intake of vitamin C
C. Employment history
D. Frequency of sexual intercourse

9. A potential adverse effect of palpating the thyroid Answer: C


gland is:

2/5
Endocrine NCLEX Questions
Study online at https://quizlet.com/_221wbp
A. Carotid artery obstruction
B. Damage to the cricoid cartilage
C. Release of excessive thyroid hormone
D. Hoarseness from pressure on the laryngeal nerve

10. The normal response to increased serum osmolality Answer: B


is the release of:

A. Aldosterone from the adrenal cortex, which stimu-


lates
sodium excretion by the kidney.
B. ADH from the posterior pituitary gland, which stim-
ulates the kidney to reabsorb water.
C. Mineralocorticoids from the adrenal gland, which
stimulates
the kidney to excrete potassium.
D. Calcitonin from the thyroid gland, which increases
bone
resorption and decreases serum calcium levels.

11. Following a hypophysectomy for acromegaly, postop- A


erative nursing care should focus on:

A. Frequent monitoring of serum and urine osmolarity


B. Parenteral administration of a GH-receptor antag-
onist
C. Keeping the patient in a recumbent position at all
times
D. Patient education regarding the need for lifelong
ACTH, TSH,
FSH, LH hormone replacement

12. The health care provider prescribes Levothyroxine for Answer: C


a patient with
hypothyroidism. Following teaching regarding this
medication, the nurse determines that further teach-
ing is needed when the patient says:

A. "I can expect the medication dose may need to be

3/5
Endocrine NCLEX Questions
Study online at https://quizlet.com/_221wbp
increased"
B. "I can expect to return to normal function with the
use of this drug"
C. "I will only need to take this medication until my
symptoms are
improved"
D. I will report any chest pain or difficulty breathing
to the doctor
right away"

13. Following thyroid surgery, the nurse suspects dam- Answer: D


age or removal of the parathyroid glands when the
patient develops:

A. Muscle weakness and weight loss


B. Hyperthermia and severe tachycardia
C. Hypertension and difficulty swallowing
D. Laryngeal stridor and tingling in the hands and feet

14. Manifestations of endocrine problems in the old- Answer: B and D


er adult that are commonly attributed to the aging
process are: (Select all that apply)

A. tremors
B. fatigue
C. fluid retention
D. mental impairment

15. The nurse is caring for a patient with Grave's disease A and D
and assessment reveals exophthalmos. Which of the
following interventions are indicated to prevent injury
to the eye: (Select all that apply)

A. Elevate HOB
B. Apply eye patches during sleep
C. Have patient blink frequently
D. Lubricating eyedrops

16. Important Nursing interventions when caring for a B and D


patient with Cushing syndrome include (select all that
4/5
Endocrine NCLEX Questions
Study online at https://quizlet.com/_221wbp
apply)

A. restricting protein intake.


B. monitoring blood glucose levels.
C. administering medication in equal doses.
D. protecting patient from exposure to infection.

17. The nurse teaches the patient that the best time to C
take corticosteroids for replacement purposes is

A. once a day at bedtime.


B. every other day on awakening.
C. on arising and in the late afternoon.
D. at consistent intervals every 6-8 hours.

5/5
Endocrine NCLEX questions
Study online at https://quizlet.com/_9oobst

1. 1. An agitated, confused fe- B.


male client arrives in the To reverse hypoglycemia, the American Di-
emergency department. Her abetes Association recommends ingesting
history includes type 1 dia- 10 to 15 g of a simple carbohydrate, such
betes mellitus, hypertension, as three to five pieces of hard candy, two to
and angina pectoris. Assess- three packets of sugar (4 to 6 tsp), or 4 oz of
ment reveals pallor, diaphore- fruit juice. If necessary, this treatment can be
sis, headache, and intense repeated in 15 minutes. Ingesting only 2 to
hunger. A stat blood glucose 5 g of a simple carbohydrate may not raise
sample measures 42 mg/dl, the blood glucose level sufficiently. Ingesting
and the client is treated for more than 15 g may raise it above normal,
an acute hypoglycemic re- causing hyperglycemia.
action. After recovery, nurse
Lily teaches the client to treat
hypoglycemia by ingesting:
a. 2 to 5 g of a simple carbo-
hydrate.
b. 10 to 15 g of a simple car-
bohydrate.
c. 18 to 20 g of a simple car-
bohydrate.
d. 25 to 30 g of a simple car-
bohydrate.

2. 2. A female adult client with A.


a history of chronic hyper- Poorly controlled hyperparathyroidism may
parathyroidism admits to be- cause an elevated serum calcium level. This,
ing noncompliant. Based on in turn, may diminish calcium stores in the
initial assessment findings, bone, causing bone demineralization and
nurse Julia formulates the setting the stage for pathologic fractures
nursing diagnosis of Risk and a risk for injury. Hyperparathyroidism
for injury. To complete the doesn't accelerate the metabolic rate. A
nursing diagnosis statement decreased thyroid hormone level, not an
for this client, which "re- increased parathyroid hormone level, may
lated-to" phrase should the cause edema and dry skin secondary to fluid
nurse add? infiltration into the interstitial spaces. Hyper-
a. Related to bone demineral- parathyroidism causes hypercalcemia, not
ization resulting in patholog- hypocalcemia; therefore, it isn't associated
ic fractures with tetany.
1 / 45
Endocrine NCLEX questions
Study online at https://quizlet.com/_9oobst
b. Related to exhaustion sec-
ondary to an accelerated
metabolic rate
c. Related to edema and dry
skin secondary to fluid in-
filtration into the interstitial
spaces
d. Related to tetany sec-
ondary to a decreased serum
calcium level

3. 3. Nurse John is assigned D.


to care for a postoperative The nurse should refer this client to a sex
male client who has diabetes counselor or other professional. Making ap-
mellitus. During the assess- propriate referrals is a valid part of planning
ment interview, the client re- the client's care. The nurse doesn't normally
ports that he's impotent and provide sex counseling.
says he's concerned about
its effect on his marriage.
In planning this client's care,
the most appropriate inter-
vention would be to:
a. Encourage the client to
ask questions about personal
sexuality.
b. Provide time for privacy.
c. Provide support for the
spouse or significant other.
d. Suggest referral to a sex
counselor or other appropri-
ate professional.

4. 4. During a class on exercise B.


for diabetic clients, a female Diabetic clients must exercise at least three
client asks the nurse educa- times a week to meet the goals of planned
tor how often to exercise. The exercise — lowering the blood glucose level,
nurse educator advises the reducing or maintaining the proper weight,
clients to exercise how often increasing the serum high-density lipopro-
to meet the goals of planned tein level, decreasing serum triglyceride lev-

2 / 45
Endocrine NCLEX questions
Study online at https://quizlet.com/_9oobst
exercise? els, reducing blood pressure, and minimiz-
a. At least once a week ing stress. Exercising once a week wouldn't
b. At least three times a week achieve these goals. Exercising more than
c. At least five times a week three times a week, although beneficial,
d. Every day would exceed the minimum requirement.

5. 5. Nurse Oliver should expect B.


a client with hypothyroidism Hypothyroidism (myxedema) causes facial
to report which health con- puffiness, extremity edema, and weight gain.
cerns? Signs and symptoms of hyperthyroidism
a. Increased appetite and (Graves' disease) include an increased ap-
weight loss petite, weight loss, nervousness, tremors,
b. Puffiness of the face and and thyroid gland enlargement (goiter).
hands
c. Nervousness and tremors
d. Thyroid gland swelling

6. 6. A female client with hy- C.


pothyroidism (myxedema) is Levothyroxine, a synthetic thyroid hormone,
receiving levothyroxine (Syn- is given to a client with hypothyroidism to
throid), 25 mcg P.O. daily. simulate the effects of thyroxine. Adverse
Which finding should nurse effects of this agent include tachycardia. The
Hans recognize as an ad- other options aren't associated with levothy-
verse drug effect? roxine.
a. Dysuria
b. Leg cramps
c. Tachycardia
d. Blurred vision

7. 7. A 67-year-old male client D.


has been complaining of Hyperparathyroidism is most common in
sleeping more, increased uri- older women and is characterized by bone
nation, anorexia, weakness, pain and weakness from excess parathy-
irritability, depression, and roid hormone (PTH). Clients also exhibit
bone pain that interferes with hypercaliuria-causing polyuria. While clients
her going outdoors. Based on with diabetes mellitus and diabetes insipidus
these assessment findings, also have polyuria, they don't have bone
nurse Richard would suspect pain and increased sleeping. Hypoparathy-
which of the following disor-

3 / 45
Endocrine NCLEX questions
Study online at https://quizlet.com/_9oobst
ders? roidism is characterized by urinary frequen-
a. Diabetes mellitus cy rather than polyuria.
b. Diabetes insipidus
c. Hypoparathyroidism
d. Hyperparathyroidism

8. 8. When caring for a male A.


client with diabetes in- Because diabetes insipidus results from de-
sipidus, nurse Juliet expects creased antidiuretic hormone (vasopressin)
to administer: production, the nurse should expect to ad-
a. vasopressin (Pitressin minister synthetic vasopressin for hormone
Synthetic). replacement therapy. Furosemide, a diuret-
b. furosemide (Lasix). ic, is contraindicated because a client with
c. regular insulin. diabetes insipidus experiences polyuria. In-
d. 10% dextrose. sulin and dextrose are used to treat diabetes
mellitus and its complications, not diabetes
insipidus.

9. 9. The nurse is aware that the D.


following is the most com- An autonomous aldosterone-producing
mon cause of hyperaldos- adenoma is the most common cause of hy-
teronism? peraldosteronism. Hyperplasia is the sec-
a. Excessive sodium intake ond most frequent cause. Aldosterone se-
b. A pituitary adenoma cretion is independent of sodium and potas-
c. Deficient potassium intake sium intake as well as of pituitary stimula-
d. An adrenal adenoma tion.

10. 10. A male client with type C.


1 diabetes mellitus has a The glycosylated Hb test provides an ob-
highly elevated glycosylated jective measure of glycemic control over
hemoglobin (Hb) test result. a 3-month period. The test helps identify
In discussing the result with trends or practices that impair glycemic con-
the client, nurse Sharmaine trol, and it doesn't require a fasting peri-
would be most accurate in od before blood is drawn. The nurse can't
stating: conclude that the result occurs from poor
a. "The test needs to be re- dietary management or inadequate insulin
peated following a 12-hour coverage.
fast."
b. "It looks like you aren't fol-

4 / 45
Endocrine NCLEX questions
Study online at https://quizlet.com/_9oobst
lowing the prescribed diabet-
ic diet."
c. "It tells us about your sugar
control for the last 3 months."
d. "Your insulin regimen
needs to be altered signifi-
cantly."

11. 11. Following a unilateral A.


adrenalectomy, nurse Bet- Muscle weakness, bradycardia, nausea, di-
ty would assess for hyper- arrhea, and paresthesia of the hands, feet,
kalemia shown by which of tongue, and face are findings associated
the following? with hyperkalemia, which is transient and
a. Muscle weakness occurs from transient hypoaldosteronism
b. Tremors when the adenoma is removed. Tremors, di-
c. Diaphoresis aphoresis, and constipation aren't seen in
d. Constipation hyperkalemia.

12. 12. Nurse Louie is develop- A.


ing a teaching plan for a ADH is the hormone clients with diabetes
male client diagnosed with di- insipidus lack. The client's TSH, FSH, and
abetes insipidus. The nurse LH levels won't be affected.
should include information
about which hormone lack-
ing in clients with diabetes in-
sipidus?
a. antidiuretic hormone
(ADH).
b. thyroid-stimulating hor-
mone (TSH).
c. follicle-stimulating hor-
mone (FSH).
d. luteinizing hormone (LH).

13. 13. Early this morning, a fe- B.


male client had a subtotal thy- Thyroid crisis usually occurs in the first
roidectomy. During evening 12 hours after thyroidectomy and caus-
rounds, nurse Tina assesses es exaggerated signs of hyperthyroidism,
the client, who now has nau- such as high fever, tachycardia, and ex-

5 / 45
Endocrine NCLEX questions
Study online at https://quizlet.com/_9oobst
sea, a temperature of 105° F treme restlessness. Diabetic ketoacidosis
(40.5° C), tachycardia, and ex- is more likely to produce polyuria, polydip-
treme restlessness. What is sia, and polyphagia; hypoglycemia, to pro-
the most likely cause of these duce weakness, tremors, profuse perspi-
signs? ration, and hunger. Tetany typically caus-
a. Diabetic ketoacidosis es uncontrollable muscle spasms, stridor,
b. Thyroid crisis cyanosis, and possibly asphyxia.
c. Hypoglycemia
d. Tetany

14. 14. For a male client with hy- C.


perglycemia, which assess- In hyperglycemia, urine osmolarity (the
ment finding best supports measurement of dissolved particles in the
a nursing diagnosis of Defi- urine) increases as glucose particles move
cient fluid volume? into the urine. The client experiences gluco-
a. Cool, clammy skin suria and polyuria, losing body fluids and ex-
b. Distended neck veins periencing fluid volume deficit. Cool, clammy
c. Increased urine osmolarity skin; distended neck veins; and a decreased
d. Decreased serum sodium serum sodium level are signs of fluid volume
level excess, the opposite imbalance.

15. 15. When assessing a male D.


client with pheochromocy- Pheochromocytoma, a tumor of the adren-
toma, a tumor of the adrenal al medulla that secretes excessive cate-
medulla that secretes exces- cholamine, causes hypertension, tachycar-
sive catecholamine, nurse dia, hyperglycemia, hypermetabolism, and
April is most likely to detect: weight loss. It isn't associated with the other
a. a blood pressure of 130/70 options.
mm Hg.
b. a blood glucose level of
130 mg/dl.
c. bradycardia.
d. a blood pressure of 176/88
mm Hg.

16. 16. A male client is admit- C.


ted for treatment of the syn- To reduce water retention in a client with the
drome of inappropriate an- SIADH, the nurse should restrict fluids. Ad-
tidiuretic hormone (SIADH). ministering fluids by any route would further

6 / 45
Endocrine NCLEX questions
Study online at https://quizlet.com/_9oobst
Which nursing intervention is increase the client's already heightened fluid
appropriate? load.
a. Infusing I.V. fluids rapidly
as ordered
b. Encouraging increased
oral intake
c. Restricting fluids
d. Administering glu-
cose-containing I.V. fluids as
ordered

17. 17. A female client has a A.


serum calcium level of 7.2 This client's serum calcium level indi-
mg/dl. During the physical cates hypocalcemia, an electrolyte imbal-
examination, nurse Noah ex- ance that causes Trousseau's sign (car-
pects to assess: popedal spasm induced by inflating the
a. Trousseau's sign. blood pressure cuff above systolic pres-
b. Homans' sign. sure). Homans' sign (pain on dorsiflexion
c. Hegar's sign. of the foot) indicates deep vein thrombosis.
d. Goodell's sign. Hegar's sign (softening of the uterine isth-
mus) and Goodell's sign (cervical softening)
are probable signs of pregnancy.

18. 18. Which outcome indicates A.


that treatment of a male client Diabetes insipidus is characterized by
with diabetes insipidus has polyuria (up to 8 L/day), constant thirst,
been effective? and an unusually high oral intake of fluids.
a. Fluid intake is less than Treatment with the appropriate drug should
2,500 ml/day. decrease both oral fluid intake and urine
b. Urine output measures output. A urine output of 200 ml/hour indi-
more than 200 ml/hour. cates continuing polyuria. A blood pressure
c. Blood pressure is 90/50 of 90/50 mm Hg and a heart rate of 126
mm Hg. beats/minute indicate compensation for the
d. The heart rate is 126 continued fluid deficit, suggesting that treat-
beats/minute. ment hasn't been effective.

19. 19. Jemma, who weighs 210 A.


lb (95 kg) and has been di- Acromegaly, which is caused by a pituitary
agnosed with hyperglycemia tumor that releases excessive growth hor-

7 / 45
Endocrine NCLEX questions
Study online at https://quizlet.com/_9oobst
tells the nurse that her hus- mone, is associated with hyperglycemia,
band sleeps in another room hypertension, diaphoresis, peripheral neu-
because her snoring keeps ropathy, and joint pain. Enlarged hands and
him awake. The nurse no- feet are related to lateral bone growth, which
tices that she has large hands is seen in adults with this disorder. The
and a hoarse voice. Which accompanying soft tissue swelling causes
of the following would the hoarseness and often sleep apnea. Type 1
nurse suspect as a possible diabetes is usually seen in children, and
cause of the client's hyper- newly diagnosed persons are usually very
glycemia? ill and thin. Hypothyroidism isn't associated
a. Acromegaly with hyperglycemia, nor is growth hormone
b. Type 1 diabetes mellitus deficiency.
c. Hypothyroidism
d. Deficient growth hormone

20. 20. Nurse Kate is provid- D.


ing dietary instructions to To control hypoglycemic episodes, the nurse
a male client with hypo- should instruct the client to consume a
glycemia. To control hypo- low-carbohydrate, high-protein diet, avoid
glycemic episodes, the nurse fasting, and avoid simple sugars. Increasing
should recommend: saturated fat intake and increasing vitamin
a. Increasing saturated fat in- supplementation wouldn't help control hypo-
take and fasting in the after- glycemia.
noon.
b. Increasing intake of vita-
mins B and D and taking iron
supplements.
c. Eating a candy bar if
light-headedness occurs.
d. Consuming a low-carbohy-
drate, high-protein diet and
avoiding fasting.

21. 21. An incoherent female C.


client with a history of hy- Severe hypothyroidism may result in
pothyroidism is brought to myxedema coma, in which a drastic drop
the emergency department in the metabolic rate causes decreased vi-
by the rescue squad. Phys- tal signs, hypoventilation (possibly leading
ical and laboratory find- to respiratory acidosis), and nonpitting ede-

8 / 45
Endocrine NCLEX questions
Study online at https://quizlet.com/_9oobst
ings reveal hypothermia, hy- ma. Thyroid storm is an acute complica-
poventilation, respiratory aci- tion of hyperthyroidism. Cretinism is a form
dosis, bradycardia, hypoten- of hypothyroidism that occurs in infants.
sion, and nonpitting ede- Hashimoto's thyroiditis is a common chronic
ma of the face and pretib- inflammatory disease of the thyroid gland in
ial area. Knowing that these which autoimmune factors play a prominent
findings suggest severe hy- role.
pothyroidism, nurse Libby
prepares to take emergency
action to prevent the poten-
tial complication of:
a. Thyroid storm.
b. Cretinism.
c. myxedema coma.
d. Hashimoto's thyroiditis.

22. 22. A male client with type B.


1 diabetes mellitus asks the Oral antidiabetic agents are only effective in
nurse about taking an oral an- adult clients with type 2 diabetes. Oral antidi-
tidiabetic agent. Nurse Jack abetic agents aren't effective in type 1 dia-
explains that these medica- betes. Pregnant and lactating women aren't
tions are only effective if the prescribed oral antidiabetic agents because
client: the effect on the fetus is uncertain.
a. prefers to take insulin oral-
ly.
b. has type 2 diabetes.
c. has type 1 diabetes.
d. is pregnant and has type 2
diabetes.

23. 23. When caring for a fe- A.


male client with a history Sulfisoxazole and other sulfonamides are
of hypoglycemia, nurse Ruby chemically related to oral antidiabetic agents
should avoid administering a and may precipitate hypoglycemia. Mexile-
drug that may potentiate hy- tine, an antiarrhythmic, is used to treat re-
poglycemia. Which drug fits fractory ventricular arrhythmias; it doesn't
this description? cause hypoglycemia. Prednisone, a corti-
a. sulfisoxazole (Gantrisin) costeroid, is associated with hyperglycemia.
b. mexiletine (Mexitil)

9 / 45
Endocrine NCLEX questions
Study online at https://quizlet.com/_9oobst
c. prednisone (Orasone) Lithium may cause transient hyperglycemia,
d. lithium carbonate (Litho- not hypoglycemia.
bid)

24. 24. After taking glipizide (Glu- B.


cotrol) for 9 months, a male Many clients (25% to 60%) with secondary
client experiences secondary failure respond to a different oral antidiabet-
failure. Which of the following ic agent. Therefore, it wouldn't be appropri-
would the nurse expect the ate to initiate insulin therapy at this time.
physician to do? However, if a new oral antidiabetic agent is
a. Initiate insulin therapy. unsuccessful in keeping glucose levels at
b. Switch the client to a differ- an acceptable level, insulin may be used in
ent oral antidiabetic agent. addition to the antidiabetic agent.
c. Prescribe an additional oral
antidiabetic agent.
d. Restrict carbohydrate in-
take to less than 30% of the
total caloric intake.

25. 25. During preoperative D.


teaching for a female client To prevent undue pressure on the surgi-
who will undergo subto- cal incision after subtotal thyroidectomy, the
tal thyroidectomy, the nurse nurse should advise the client to avoid hy-
should include which state- perextending the neck. The client may el-
ment? evate the head of the bed as desired and
a. "The head of your bed must should perform deep breathing and cough-
remain flat for 24 hours after ing to help prevent pneumonia. Subtotal thy-
surgery." roidectomy doesn't affect swallowing.
b. "You should avoid deep
breathing and coughing after
surgery."
c. "You won't be able to swal-
low for the first day or two."
d. "You must avoid hyper-
extending your neck after
surgery."

26. What is a hormone secreted . C


from the posterior lobe of the ADH is secreted from the posterior pituitary.

10 / 45
Endocrine NCLEX questions
Study online at https://quizlet.com/_9oobst
pituitary gland? LH comes from the anterior pituitary, MSH
Answers: from the intermediate. GnRH is released
A. LH from the hypothalamus.
B. MSH
C. ADH
D. GnRH

27. An indication of Chvostek' .A


sign is: Twitching of the lips after tapping the face in
Answers: the right place is an indication of Chvostek's
A. Twitching of the lips after sign and a sign of hypocalcaemia. Spasms
tapping the face of the hand are associated with Trousseau's
B. Elevated blood sugar after sign.
glucose infusion
C. Inability to hold one's arms
straight
D. Spasms of the hand after
blood circulation is cut off

28. A 26 year old female client B


presents with the symptom PCOS is well known to cause hormonal ir-
of unwanted facial hair. What regularities in women which can result in
of the following conditions is hair growth.
most likely?
Answers:
A. Graves' disease
B. PCOS
C. Hyperthyroidism
D. Addison's disease

29. In explaining the condition to D


a client, a nurse would say Cushing's syndrome is caused by elevat-
that Cushing's syndrome is ed levels of cortisol. Glucocorticoids tend to
caused primarily by: cause this.
Answers:
A. Low levels of glucocorti-
coids
B. Excess secretion of sodi-
um

11 / 45
Endocrine NCLEX questions
Study online at https://quizlet.com/_9oobst
C. Autoimmunity in the pan-
creas
D. Elevated levels of cortisol

30. Which of the following symp- B


toms is not typical of Cush- Cushing's syndrome tends to produce rapid
ing's syndrome? weight gain, not weight loss.
Answers:
A. Osteoporosis
B. Weight loss
C. Diabetes
D. Mood instability

31. Which of the following would A


be an indication of Androgen Androgen Insensitivity Syndrome is when
Insensitivity Syndrome? the body does not respond to androgens
Answers: such as testosterone. This can result in ge-
A. A 33 year old woman with netic males being born with the appearance
a karyotype of XY of women.
B. A 16 year old male with re-
duced kidney function
C. Failure to respond to corti-
sol therapy
D. Several pregnancies all of
which ended in miscarriages

32. A client presents with C


hypocalcemia, hyperphos- Hypoparathyroidism often leads to the
phatemia, muscle cramps, symptoms mentioned. Conn's syndrome is
and positive Trosseau's sign. an aldosterone-producing adenoma.
What diagnosis does this
support?
Answers:
A. Diabetes insipidus
B. Conn's syndrome
C. Hypoparathyroidism
D. Acromegaly

33. A client with Graves' disease . D


experiences a thyroid storm Propylthiouracil is a commonly used med-
12 / 45
Endocrine NCLEX questions
Study online at https://quizlet.com/_9oobst
and has tachycardia and hy- ication for treating hyperthyroidism. Lev-
pertension. What medication ofloxacin is an antibiotic, chlorothiazide is a
is most likely to be used? diuretic, and Percocet a painkiller.
Answers:
A. Levofloxcin
B. Chlorothiazide
C. Percocet
D. Propylthiouracil

34. Which of the following state- .B


ments by a client with Type II While an HbA1C level of 5.5% would be
Diabetes indicates the need below the threshold for diabetes, it is an
for further education? unrealistic target. Data has shown that trying
Answers: to lower the HbA1C level too much can lead
A. I should avoid hot tubs to an increase in complications.
B. I should aim for an HbA1C
level of 5.5%
C. I may need insulin at times
D. My life expectancy is likely
reduced by 10 years

35. In educating a client, the A


nurse is likely to explain the Hashimoto's disease is caused by autoim-
following is the cause of munity to the thyroid gland, often involving
Hashimoto's disease: antibodies.
Answers:
A. Antibodies attacking the
thyroid gland
B. Inflammation in the kid-
neys
C. An adenocarcinoma in the
brain
D. Overactivation of the pitu-
itary gland

36. Acromegaly is most frequent- . A: Acromegaly results from benign tumors


ly diagnosed in: on the pituitary gland that produce exces-
a. Middle-aged adults sive amounts of growth hormone. Although
b. Newborns symptoms may present at any age, the

13 / 45
Endocrine NCLEX questions
Study online at https://quizlet.com/_9oobst
c. Children ages 2 to 5 diagnosis generally occurs in middle-aged
d. Adults age 65 and older persons. Untreated, the consequences of
acromegaly include

37. 2. Grave's disease is: type 2 diabetes, hypertension and increased


a. The most common cause of risk of cardiovascular disease, arthritis and
hypothyroidism colon polyps.
b. The most common cause of 2. C: Grave's disease is an autoimmune
hyperparathyroidism disorder characterized by an enlarged thy-
c. The most common cause of roid gland and overproduction of thyroid hor-
hyperthyroidism mones producing symptoms of hyperthy-
d. The most common cause of roidism such as rapid heartbeat, heat intol-
adrenal insufficiency erance, agitation or irritability, weight loss,
and trouble sleeping. It usually presents in
persons age 20 to 40 and it is much more
common in women than in men.

38. 3. Symptoms of Grave's oph- 3. C: Grave's ophthalmopathy is an inflam-


thalmopathy include all of the mation of tissue behind the eye causing
following except: the eyeballs to bulge. In addition to the
a. Bulging eyeballs above-mentioned symptoms, Grave's oph-
b. Dry, irritated eyes and puffy thalmopathy may cause pressure or pain in
eyelids the eyes, double vision, and trouble moving
c. Cataracts the eyes. About one-quarter of persons with
d. Light sensitivity Grave's disease develop Grave's ophthal-
mopathy. The condition is frequently self-lim-
iting, resolving without treatment over the
course of a year or two.

39. 4. An ACTH stimulation test is 4. B: The ACTH stimulation test measures


commonly used to diagnose: blood and urine cortisol before and after
a. Grave's disease injection of ACTH. Persons with chronic
b. Adrenal insufficiency and adrenal insufficiency or Addison's disease
Addison's disease generally do not respond with the expect-
c. Cystic fibrosis ed increase in cortisol levels. An abnormal
d. Hashimoto's disease ACTH stimulation test may be followed with
a CRH stimulation test to pinpoint the cause
of adrenal insufficiency.

40.
14 / 45
Endocrine NCLEX questions
Study online at https://quizlet.com/_9oobst
5. All of the following are 5. D: Cushing's syndrome also may cause
symptoms of Cushing's syn- fragile, thin skin prone to bruises and stretch
drome except: marks on the abdomen and thighs as well
a. Severe fatigue and weak- as excessive thirst and urination and mood
ness changes such as depression and anxiety.
b. Hypertension and elevated Women who suffer from high levels of corti-
blood glucose sol often have irregular menstrual cycles or
c. A protruding hump be- amenorrhea and present with hair on their
tween the shoulders faces, necks, chests, abdomens, and thighs.
d. Hair loss

41. 6. Which of the follow- 6. D: Cushing's syndrome is a form of hy-


ing conditions is caused by percortisolism. Risk factors for Cushing's
long-term exposure to high syndrome are obesity, diabetes, and hyper-
levels of cortisol? tension. Cushing's syndrome is most fre-
a. Addison's disease quently diagnosed in persons ages 20 to 50
b. Crohn's disease who have characteristic round faces, upper
c. Adrenal insufficiency body obesity, large necks, and relatively thin
d. Cushing's syndrome limbs.

42. 7. A "sweat test" or newborn 7. A: Cystic fibrosis is the most common


screening may be used to de- inherited fatal disease of children and young
tect: adults in the United States. Cystic fibrosis is
a. Cystic fibrosis usually diagnosed by the time an affected
b. Adrenal insufficiency child is three years old. Often, the only signs
c. Grave's disease are a persistent cough, a large appetite but
d. Hypothyroidism poor weight gain, an extremely salty taste
to the skin, and large, foul-smelling bowel
movements. A simple sweat test is currently
the standard diagnostic test. The test mea-
sures the amount of salt in the sweat; ab-
normally high levels are the hallmark of the
disorder.

43. 8. Hashimoto's disease is: 8. A: Hashimoto's disease is the most com-


a. Chronic inflammation of mon cause of hypothyroidism. It is an au-
the thyroid gland toimmune disease that produces chronic
b. Diagnosed most frequently inflammation of the thyroid gland. More
in Asian-Americans and Pa- women are affected than men and it is gen-

15 / 45
Endocrine NCLEX questions
Study online at https://quizlet.com/_9oobst
cific Islanders erally diagnosed in persons ages 40 to 60.
c. A form of hyperthyroidism When treatment is indicated, synthetic T4 is
d. A rare form of hypothy- administered.
roidism

44. 9. Persons at increased risk 9. B: Along with the above-mentioned


of developing Hashimoto's groups, persons with type 1 diabetes and
disease include all of the fol- persons suffering from pernicious anemia
lowing except: (insufficient vitamin b12) are at increased
a. Persons with vitiligo risk of developing Hashimoto's disease. Be-
b. Asian-Americans cause it tends to run in families, there is
c. Persons with rheumatoid likely a genetic susceptibility as well. Envi-
arthritis ronmental factors such as excessive iodine
d. Persons with Addison's consumption and selected drugs also have
disease been implicated as potential risk factors.

45. 10. All of the following state- 10. C: Although weight gain may be a symp-
ments about Hashimoto's tom of Hashimoto's disease, the majority of
disease are true except: obese people have normal thyroid function;
a. Many patients are entirely rarely is thyroid disorder the sole cause of
asymptomatic obesity. Other symptoms of Hashimoto's dis-
b. Not all patients become hy- ease include fatigue, cold intolerance, joint
pothyroid pain, myalgias, constipation, dry hair, skin
c. Most cases of obesity are and nails, impaired fertility, slow heart rate,
attributable to Hashimoto's and depression.
disease
d. Hypothyroidism may be
subclinical

46. 11. The most common benign 11. B: Prolactinomas can cause symptoms
tumor of the pituitary gland is by releasing excessive amounts of prolactin
a: into the blood or mechanically by pressing
a. Glioma on surrounding tissues. In women, symp-
b Prolactinoma toms may include menstrual irregularities
c. Carcinoid tumor and infertility; in men erectile dysfunction
d. Islet cell tumor and libido may be impaired.

47. 12. Symptoms of polycys- 12. D: In addition to the above-mentioned


tic ovarian syndrome (PCOS) symptoms, PCOS may cause menstrual ir-
may include all of the follow- regularities, thinning hair or male-pattern
16 / 45
Endocrine NCLEX questions
Study online at https://quizlet.com/_9oobst
ing except: baldness, thick skin or dark patches of skin
a. Pelvic pain and excessive hair growth on the face,
b. Acne, oily skin, and dan- chest, abdomen, thumbs and toes.
druff
c. Infertility
d. Weight Loss

48. 13. Women with PCOS are at 13. A: Women with PCOS produce exces-
increased risk for all of the sive amounts of androgens and do not re-
following except: lease ova during ovulation, which serious-
a. Pregnancy ly compromises their ability to conceive.
b. Diabetes Although women with PCOS can become
c. Cardiovascular disease pregnant, often by using assistive reproduc-
d. Metabolic syndrome tive technology, they are at increased risk for
miscarriage.

49. 14. All of the following organs 14. B: Multiple endocrine neoplasia type
may be affected by multiple 1, also known as Werner's syndrome, is
endocrine neoplasia type 1 a heritable disorder that causes tumors in
except: endocrine glands and the duodenum. Al-
a. Parathyroid glands though the tumors associated with multiple
b. Kidneys endocrine neoplasia type 1 are generally
c. Pancreas and Duodenum benign, they can produce symptoms chemi-
d. Pituitary gland cally by releasing excessive amounts of hor-
mones or mechanically by pressing on adja-
cent tissue.

50. 15. What is the treatment for 15. C: When hyperparathyroidism requires
hyperparathyroidism? treatment, surgery is the treatment of choice
a. Synthetic thyroid hormone and is considered curative for 95% of cas-
b. Desiccated thyroid hor- es. Because untreated hyperparathyroidism
mone may elevate blood and urine levels of cal-
c. Surgical removal of the cium and deplete phosphorus, bones and
glands teeth may lose the minerals needed to re-
d. Calcium and phosphate main strong.

51. 16. The most common causes 16. D: Declining pulmonary function is a
of death in people with cystic hallmark of cystic fibrosis. Drugs such as
fibrosis is: Pulmozyme (dornase alfa) and Zithromax
a. Dehydration (azithromycin) can slow the progression of
17 / 45
Endocrine NCLEX questions
Study online at https://quizlet.com/_9oobst
b. Opportunistic infection lung disease and mechanical physical thera-
c. Lung cancer py devices help CF patients to breathe more
d. Respiratory failure easily by loosening and dislodging mucus.
For some patients with severe lung damage,
lung transplantation is a treatment option.

52. 17. Untreated hyperthy- 17. C: In addition to the above-men-


roidism during pregnancy tioned complications of uncontrolled hyper-
may result in all of the follow- thyroidism in pregnancy, expectant mothers
ing except: may suffer congestive heart failure and thy-
a. Premature birth and mis- roid storm, which is life-threatening thyrotox-
carriage icosis with symptoms that include agitation,
b. Low birthweight confusion, tachycardia, shaking, sweating,
c. Autism diarrhea, fever, and restlessness.
d. Preeclampsia

53. 18. Short stature and un- 18. D: Turner syndrome results from a chro-
developed ovaries suggest mosomal abnormality and occurs in an es-
which of the following disor- timated 1 in 2,500 female births. It occurs
ders: more frequently in preterm pregnancies. Af-
a. Polycystic ovarian syn- fected women are shorter than average and
drome are infertile because they lack ovarian func-
b. Prolactinoma tion. They also may have webbed necks,
c. Grave's disease broad chests, arms that turn out from the
d. Turner syndrome elbow, lymphedema of the hands and feet
and skeletal, cardiac, and renal problems.

54. 19. Endocrine disorders may 19. D: Endocrine function may be influ-
be triggered by all of the fol- enced by myriad factors. In addition to the
lowing except: above-mentioned, there is evidence that ex-
a. Stress posure to naturally occurring and man-made
b. Infection endocrine disruptors such as tributyltin,
c. Chemicals in the food certain bioaccumulating chlorinated com-
chain and environment pounds, and phytoestrogens is widespread
d. Cell phone use and in susceptible individuals, may trigger
endocrine disorders.

55. 20. An analysis of data from 20. D: The results of the Women's Health
the Women's Health Initiative Initiative study prompted the U.S. Food and
questioned the use of which Drug Administration (FDA) to advise against
18 / 45
Endocrine NCLEX questions
Study online at https://quizlet.com/_9oobst
therapy to prevent heart dis- using hormone therapy (estrogen-alone or
ease? estrogen-plus-progestin) to prevent heart
a. Synthetic thyroid hormone disease. When hormone replacement ther-
b. Oral contraceptives apy is used to treat moderate to severe hot
c. Weight-loss drugs flashes and symptoms of vulvar and vaginal
d. Postmenopausal hormone atrophy it should used at the lowest doses
replacement therapy for the shortest duration needed to achieve
treatment objectives.

56. The parathyroid glands play a a


major role in regulating which
substances?
A. Calcium and Phosphorus
B. Cholride and potassium
C. Potassium and calcium
D. Sodium and potassium a.
Calcium and Phosphorus

57. The primary function of in- a


suln is to:
A. Lower blood glucose lev-
els
B. Produce melanin
C. regulate the bodys meta-
bolic rate
D. stimulate release of diges-
tive enzymes

58. a client is admitted to the hos- d


pital with a medical DX of
hyerthyroidism. When taking
a history which information
would be most significant?
A. edema, intolerance to cold,
lethargy
b. peri-orbital edema, lethar-
gy mask like face
c. weight loss, intolerance to
cold, muscle wasting

19 / 45
Endocrine NCLEX questions
Study online at https://quizlet.com/_9oobst
d. weight loss, intolerance to
heat, exophthalmos

59. Which nursing action is most b


appropriate for a client in ke-
toacidosis?
a. admin of carbs
b. admin of IV fluids
c. applying cold compress
d. giving glucagon IV

60. The nurse smells a sweet c


fruity odor on the breath of
a client admitted with DM.
This odor may be associated
with?
a. alcohol intoxication
b. insulin shock
c. ketoacidosis

61. A client asks what the pur- a


pose of the Hb A1c test is.
The nurses best explanation
would be that the test mea-
sures the average:
a. blood sugar lvl's over a
6-10 week period
b. hemoglobin lvl's over a 6 -
10 week period
c. protien lvl over a 3 month
period
d. vanillylmandelic acid lvl's

62. which of the following would c


be a nursing priority for a
client just DX with Addison's
disease?
a. avioding unnecessary ac-
tivity
b. encouraging client to wear
20 / 45
Endocrine NCLEX questions
Study online at https://quizlet.com/_9oobst
a med alert tag
c. ensuring the client is ade-
quatly hydrated
d. explaining that the client
will need life long hormone
therapy

63. A nurse is caring for a client b


in the late stage of Ketoaci-
dosis. The nurse notices that
the clients breath has a char-
acteristic fruity odor. Which
of the following substances
is responsible for the fruity
smell in the breath?
a. iodine
b. acetone
c.alcohol
d. glucose

64. A nurse is caring for a c


client with Addison's dis-
ease. Which of the follow-
ing mursing considerations
shoul dbe employed when
caring for this client?
a. avoid sodium in the clients
diet
b. monitor and protect skin
integrity
c. document the specific
gravity of urine
d. monitor increases in blood
pressure

65. A nurse is assigned to care d


for and monitor any compli-
cations in a 40 yr client with
chronic diabetes. Which of

21 / 45
Endocrine NCLEX questions
Study online at https://quizlet.com/_9oobst
the following is a macrovas-
cular complication of dia-
betes.
a. neuropathy
b. retinopathy
c. nephropathy
d. Arteriosclerosis

66. A nurse is instructing a c


50yr diabetic client about the
steps to be followed for self
admin of insulin. Which of the
following instructions should
be included in te client teach-
ing?
a. instruct client to aviod in-
jections to the abdomen
b. encourage client to always
inject insulin in the same site
c. inform client about the type
of syringe to use
d. encourage client to do ac-
tive exercise after injection

67. A nurse is preparing a diet a


plan for a 50yr with simple
goiter. Which of the follow-
ing should be included in
teh clients diet to decrease
the enlargement of he thyroid
gland?
a. iodine
b. sodium
c. potassium
d. calcium

68. A nurse is caring for a d


60yr client affected with
hypoparathyroidism. When

22 / 45
Endocrine NCLEX questions
Study online at https://quizlet.com/_9oobst
checking the lab report, the
nurse finds tht the clients cal-
cium lvl was very low. Which
of the following vitamins reg-
ulates teh calcium lvl in the
body?
a. A
b. D
c. E
d. K

69. A client presents to the emer- Replace intravenous fluids


gency room with a history
of Graves' disease. The client Induce shivering
reports having symptoms for
a few days, but has not pre- Relieve respiratory distress
viously sought or received
any additional treatment. The Administer a cooling blanket
client also reports having had
a cold a few days back. WhichRationale: Thyroid storm (also called thyroid
of the following interventions
crisis) is an extreme state of hyperthyroidism
would be appropriate to im- that is rare today because of improved diag-
plement for this client, based
nosis and treatment methods (Porth, 2005).
on the history and current When it does occur, those affected are usu-
symptoms? Select all that ap-ally people with untreated hyperthyroidism
ply. (most often Graves' disease) and people
with hyperthyroidism who have experienced
a stressor, such as an infection, trauma.
Administer aspirin The rapid increase in metabolic rate that
results from the excessive TH causes the
Replace intravenous fluids manifestations of thyroid storm. The mani-
festations include hyperthermia, with body
Induce shivering temperatures ranging from 102°F (39°C) to
106°F (41°C); tachycardia; systolic hyper-
Relieve respiratory distress tension; and gastrointestinal symptoms (ab-
dominal pain, vomiting, diarrhea). Agitation,
Administer a cooling blanket restlessness, and tremors are common, pro-
gressing to confusion, psychosis, delirium,
and seizures. The mortality rate is high.
23 / 45
Endocrine NCLEX questions
Study online at https://quizlet.com/_9oobst
Rapid treatment of thyroid storm is essential
to preserve life. Treatment includes cooling
without aspirin (which increases free TH)
or inducing shivering, replacing fluids, glu-
cose, and electrolytes, relieving respiratory
distress, stabilizing cardiovascular function,
and reducing TH synthesis and secretion. #1
is incorrect because cooling happens with-
out the use of aspirin. All of the other choices
are correct.

70. A nursing student is study- Correct Answer: "Decreased renal blood


ing for a test on care of flow and glomerular filtration rate reduces
the client with endocrine dis- the kidney's ability to excrete water, which
orders. Which of the follow- may cause hyponatremia."
ing statements demonstrates Rationale: # 1 is incorrect because deficient
an understanding of the dif- amounts of TH cause abnormalities in lipid
ference between hyperthy- metabolism with elevated serum cholesterol
roidism and hypothyroidism? and triglyceride levels. # 2 is incorrect be-
cause Graves' disease is the most com-
mon cause of hyperthyroidism, not hypothy-
"Deficient amounts of TH roidism. # 4 is incorrect because increased
cause abnormalities in amounts of TH cause an increase in cardiac
lipid metabolism, with de- output and peripheral blood flow.
creased serum cholesterol
and triglyceride levels."

"Graves' disease is the most


common cause of hypothy-
roidism."

"Decreased renal blood flow


and glomerular filtration rate
reduces the kidney's ability
to excrete water, which may
cause hyponatremia."

"Increased amounts of TH
cause a decrease in cardiac
24 / 45
Endocrine NCLEX questions
Study online at https://quizlet.com/_9oobst
output and peripheral blood
flow."

71. A Clinical Instructor is ques- "Hypoparathyroidism results in decreased


tioning a student nurse about activation of vitamin D which then results
disorders of the parathyroid in decreased absorption of calcium by the
glands. Which statement by pancreas."
the nursing student, would in- Rationale: Choices 1, 2, and 3 are all correct
dicate the need for further statements. # 4 demonstrates a need for fur-
teaching? ther teaching because hypoparathyroidism
results in decreased activation of vitamin D
which then results in decreased absorption
of calcium by the intestines, not the pan-
"Hyperparathyroidism re- creas.
sults in an increased release
of calcium and phosphorus
by bones, with resultant bone
decalcification."

"Hyperparathyroidism re-
sults in deposits in soft tis-
sues and the formation of re-
nal calculi."

"Hypoparathyroidism results
in impaired renal tubular reg-
ulation of calcium and phos-
phate."

"Hypoparathyroidism results
in decreased activation of vi-
tamin D which then results in
decreased absorption of cal-
cium by the pancreas."

72. A nurse on a general med- Mitotane is used to treat metastatic adrenal


ical-surgical unit is caring for cancer.
a client with Cushing's syn- Rationale: Mitotane directly suppresses ac-
drome. Which of the following tivity of the adrenal cortex and decreases

25 / 45
Endocrine NCLEX questions
Study online at https://quizlet.com/_9oobst
statements is correct about peripheral metabolism of corticosteroids. It
the medication regimen for is used to treat metastatic adrenal cancer.
Cushing's syndrome? # 2 is incorrect because aminogluthimide
may be administered to clients with ectopic
ACTH-secreting tumors that cannot be sur-
Mitotane is used to treat gically removed. # 3 is incorrect because
metastatic adrenal cancer. ketoconazole inhibits, not increases, cortisol
synthesis by the adrenal cortex. # 4 is incor-
Aminogluthimide may be ad- rect because somatostatin suppresses, not
ministered to clients with ec- increases, ACTH secretion.
topic ACTH-secreting tumors
before surgery is performed.

Ketoconazole increases cor-


tisol synthesis by the adrenal
cortex.

Somatostatin analog increas-


es ACTH secretion in some
clients.

73. Which of the following nurs- Instruct the client to never abruptly discon-
ing implications is most im- tinue the medication.
portant in a client being med- Rationale: The primary medical treatment of
icated for Addison's disease? Addison's disease is replacement of corti-
costeroids and mineralcorticoids, accompa-
nied by increased sodium in the diet. The
client needs to know the importance of main-
Administer oral forms of the taining a diet high is sodium and low in
drug with food to minimize its potassium. Medications should never be dis-
ulcerogenic effect. continued abruptly because crisis can en-
sue. Oral forms of the drug are given with
Monitor capillary blood glu- food in Cushing's disease.
cose for hypoglycemia in the
diabetic client.

Instruct the client to nev-


er abruptly discontinue the
medication.
26 / 45
Endocrine NCLEX questions
Study online at https://quizlet.com/_9oobst

Teach the client to consume


a diet that is high in potassi-
um, low in sodium, and high
in protein.

74. A nurse on a surgical floor Assess respiratory rate, rhythm, depth, and
is caring for a post-oper- effort.
ative client who has just Rationale: All of the above assessments
had a subtotal thyroidecto- have importance, but airway and breathing
my. Which of the following as- in a client should always be addressed first
sessments should be com- when prioritizing care. Assess for signs of
pleted first on the client? latent tetany due to calcium deficiency, in-
cluding tingling of toes, fingers, and lips;
muscular twitches; positive Chvostek's and
Assess for signs of tetany by Trousseau's signs; and decreased serum
checking for Chvostek's and calcium levels. However, tetany may occur
Trousseau's signs in 1 to 7 days after thyroidectomy so # 1 is
not the highest priority. Assessing for hem-
Assess dressing (if present) orrhage is always important, but the danger
and the area under the of hemorrhage is greatest in the first 12 to
client's neck and shoulders 24 hours after surgery, and as this client
for drainage. is immediately post operative it is not the
main concern at this time. Pain medication is
Administer analgesic pain important but according to Maslow, pain is a
medications as ordered, and psychosocial need to be addressed after a
monitor their effectiveness. physiologic need.

Assess respiratory rate,


rhythm, depth, and effort.

75. The nurse is caring for a Maintain careful use of medical and surgi-
client who is about to under- cal asepsis when providing care and treat-
go an adrenalectomy. Which ments.
of the following Preoperative Rationale: Use careful medical and surgical
interventions is most appro- asepsis when providing care and treatments
priate for this client? since Cortisol excess increases the risk of
infection. # 2 is incorrect. Nutrition should
be addressed preoperatively. Request a di-

27 / 45
Endocrine NCLEX questions
Study online at https://quizlet.com/_9oobst
etary consultation to discuss with the client
Maintain careful use of med- about a diet high in vitamins and proteins.
ical and surgical asepsis If hypokalemia exists, include foods high in
when providing care and potassium. Glucocorticoid excess increases
treatments. catabolism. Vitamins and proteins are nec-
essary for tissue repair and wound healing
Teach the client about a diet following surgery. # 3 is incorrect. Monitor
high in sodium to correct any the results of laboratory tests of electrolytes
potential sodium imbalances and glucose levels. Electrolyte and glucose
preoperatively. imbalances are corrected

Explain to the client that elec-


trolytes and glucose levels
will be measured postopera-
tively.

Teach the client how to ef-


fectively cough and deep
breathe once surgery is com-
plete.

76. The nurse is caring for a Monitor blood pressure frequently, assess-
client with pheochromocy- ing for hypertension.
toma. Which of the following Rationale: Pheochromocytomas are tumors
must be included in planning of chromaffin tissues in the adrenal medul-
the nursing care for this client
la. These tumors which are usually be-
? nign produce catecholamines (epinephrine
or norepinephrine) that stimulate the sym-
pathetic nervous system. Although many or-
Monitor blood pressure fre- gans are affected, the most dangerous ef-
quently, assessing for hyper- fects are peripheral vasoconstriction and in-
tension. creased cardiac rate and contractility with
resultant paroxysmal hypertension. Systolic
Assess only for physical blood pressure may rise to 200 to 300
stressors present. mmHg, the diastolic to 150 to 175 mmHg. #
1 is correct because the careful monitoring
Collect a random urine sam- of blood pressure is essential. Attacks are
ple. often precipitated by physical, emotional, or
environmental stimuli, so # 2 is incorrect
28 / 45
Endocrine NCLEX questions
Study online at https://quizlet.com/_9oobst
Prepare the client for because more than physical stressors are
chemotherapy to shrink the considered. This condition is life threatening
tumor. and is usually treated with surgery as the
preferred treatment. # 3 is incorrect because
it is a random sample and not a 24 hour
urine collection. Because catecholamine se-
cretion is episodic, a 24-hour urine is a bet-
ter surveillance method than serum cate-
cholamines. (Pagana & Pagana, 2002). Sur-
gical removal of the tumor(s) by adrenalecto-
my is the treatment of choice. # 4 is incorrect
because surgery would be the treatment
usually completed.

77. A client newly diagnosed with "I need to weigh myself if I think I am losing
Addison's disease is giving a or gaining weight."
return explanation of teach- The client is at risk for ineffective therapeu-
ing done by the primary tic regimen management. Clients with Addi-
nurse. Which of the following son's disease must learn to provide lifelong
statements indicates that fur- self-care that involves varied components:
ther teaching is necessary? medications, diet, and recognizing and re-
sponding to stress. Changes in lifestyle are
difficult to maintain permanently. The client
needs to take the medications on a daily ba-
"I need to increase how much sis. The client needs to perform daily weights
I drink each day." to monitor for signs of dehydration. The client
needs to maintain a diet high in sodium and
"I need to weigh myself if I low in potassium, as well as maintain an in-
think I am losing or gaining creased fluid intake. # 2 is incorrect because
weight." daily weights need to be performed instead
of weighing when a problem is suspected.
"I need to maintain a diet high
in sodium and low in potassi-
um."

"I need to take my medica-


tions each day."

78.

29 / 45
Endocrine NCLEX questions
Study online at https://quizlet.com/_9oobst
. The nurse assessing a fe- 1) A
male client with Cushing's - An increased production of androgens
syndrome would expect to that accompanies a rise in cortisol lev-
note which of the following? els with Cushing's syndrome produces hir-
sutism and acne in women. Other clinical
a) hirsutism findings of Cushing's syndrome include hy-
b) hypotension pertension caused by sodium retention, im-
c) hypoglycemia paired glucose tolerance or diabetes mel-
d) pallor litus caused by cortisol's anti-insulin effect
and ability to enhance gluconeogenesis,
and skin changes including bruising and
purplish red striae caused by protein catab-
olism.

79. 2. A nurse is admitting a client 2) C


with a diagnosis of Addison's - Blood glucose levels are low in Addison's
disease to the hospital. On disease as a result of decreased secretion of
assessment, the nurse would glucocorticoids (cortisol). Edema is absent,
expect to note which finding and aldosterone secretion is decreased so
that is a manifestation of this the client develops a deficient fluid volume.
disorder? Facial hair increases with adrenocortical hy-
perfunction. Clients with Addison's disease
a) peripheral edema develop hypotension as a result of deficient
b) excessive facial hair fluid volume. Options A, B and D are unre-
c) lower than normal blood lated to Addison's disease.
glucose level
d) high blood pressure

80. 3. A nurse is preparing to 3) A


perform an assessment on - Excessive secretion of adrenocortical hor-
a client being admitted to mones results in water and sodium re-
the hospital with a diagno- absorption, causing fluid retention. Stretch
sis of Cushing's syndrome. marks (striae) are a common feature and
When performing the assess- can result in a disturbed body image,
ment, the nurse checks for but are not significant and do not repre-
which significant manifesta- sent a life-threatening situation. Goiter is
tion of the disorder? not a manifestation of Cushing's syndrome.
Melanosis is a common manifestation asso-
a) fluid retention ciated with Addison's disease.

30 / 45
Endocrine NCLEX questions
Study online at https://quizlet.com/_9oobst
b) stretch marks
c) goiter
d) melanosis

81. 4. A clinic nurse is perform- 4) D


ing an assessment on a client - An insufficient level of thyroid hormone
who has hypothyroidism. The causes a decrease in metabolic rate and
nurse would expect to note heat production. Intolerance to cold would
which clinical manifestation? be noted. Options A, B and C are clinical
manifestations of hyperthyroidism.
a) complaints of difficulty
sleeping
b) complaints of diarrhea
c) significant weight loss
since the last clinic visit
d) complaints of intolerance
to cold weather

82. 5. A clinic nurse is perform- 5) C


ing an assessment on a client Open-ended questions allow the client to
recently diagnosed with dia- take the lead in the conversation. Options
betes mellitus. Which assess- A and B denote judgment and may block
ment question is appropriate communication. Option D allows the client
when assessing the client's to answer with a yes or no response and
degree of adaptation to this does not provide the client an opportunity to
disorder? share feelings. Option C is open-ended and
focuses on the subject of the question, the
a) you really don't think client's degree of adaptation to the disorder.
you caused your disorder, do
you?
b) your family is helping you
stick to your diet, aren't they?
c) how do you feel about your
progress?
d) are you feeling anxious?

83. . A client has been diagnosed 6) C


with goiter. The nurse looks - An enlarged thyroid gland occurs in goiter.
for documentation of which Decreased wound healing, chronic fatigue,

31 / 45
Endocrine NCLEX questions
Study online at https://quizlet.com/_9oobst
of the following in the client's and heart damage are not specifically asso-
medical record? ciated with this condition.

a) decreased wound healing


b) chronic fatigue
c) enlarged thyroid gland
d) heart damage

84. 7. A nurse is assessing 7) B


a lethargic client who was - Clients with DKA accumulate large
brought to the emergency de- amounts of ketone bodies in extracellular
partment by emergency med- fluids. A fruity odor to the breath develops
ical services and notes a due to the volatile nature of acetone. A fruity
fruity odor to the client's odor is not a manifestation associated with
breath. The nurse immediate- the conditions noted in options A, C, and D.
ly suspects that the client
has:

a) hyperglycemic hyperos-
molar nonketotic syndrome
(HHNS)
b) diabetic ketoacidosis
(DKA)
c) ethanol oxide intoxication
d) hypoglycemia

85. 8. A nurse is caring for a client 8) C


following thyroidectomy and - Hypocalcemia can develop after thyroidec-
is monitoring for complica- tomy if the parathyroid glands are acciden-
tions. Which of the following tally removed or traumatized during surgery.
if noted in the client, would If the client develops numbness and tingling
indicate a need for physician around the mouth or in the fingertips or toes,
notification? muscle spasms, or twitching, the physician
should be called immediately. A hoarse or
a) surgical pain in the neck weak voice may occur temporarily if there
area has been unilateral injury to the laryngeal
b) voice hoarseness nerve during surgery. Pain is expected in
c) numbness and tingling the postoperative period. Calcium gluconate
ampules should be available at the bedside,

32 / 45
Endocrine NCLEX questions
Study online at https://quizlet.com/_9oobst
around the mouth and the client should have a patent intra-
d) weakness of the voice venous (IV) line in the event that hypocal-
cemic tetany occurs

86. 9. A nurse is monitoring 9) C


a client for complications Temporary hoarseness and a weak voice
following thyroidectomy. The may occur if there has been unilateral in-
nurse notes that the client's jury to the laryngeal nerve during surgery. If
voice is very hoarse, and hoarseness or a weak voice is present, the
the client is concerned about client is reassured that the problem will prob-
the hoarseness and asks the ably subside in a few days. Unnecessary
nurse about it. The nurse talking is discouraged to minimize hoarse-
makes which response to al- ness. The statements in options A, B, and D
leviate the client's concern? will not alleviate the client's concern.

a) hoarseness and weak


voice indicate permanent
damage to the nerves
b) this complication is ex-
pected
c) this problem is temporary
and will probably subside in
a few days
d) it is best that you not talk at
all until the problem is further
evaluated

87. 10. A nurse provides dietary 10) C


instructions to a client with - The client with hyperparathyroidism should
a diagnosis of hyperparathy- consume at least 3000 mL of fluid per day.
roidism. Which statement by Measures to prevent dehydration are neces-
the client indicates the need sary because dehydration increases serum
for further instructions? calcium levels and promotes the formation
of renal stones. Cranberry juice and prune
a) I need to drink 3000 ml of juice help make the urine more acidic. A high
fluid per day urinary acidity helps prevent renal stone for-
b) I should drink cranberry mation because calcium is more soluble in
juice daily acidic urine than in alkaline urine. Clients
c) I should eat foods high in should be on a low-calcium, low-vitamin D

33 / 45
Endocrine NCLEX questions
Study online at https://quizlet.com/_9oobst
calcium diet. High-fiber foods are important to pre-
d) I should eat foods high in vent constipation and fecal impaction result-
fiber ing from the hypercalcemia that occurs with
this disorder.

88. 16. A nurse is caring for a 16) D


client with hyperthyroidism - The client with hyperthyroidism is usual-
and is instructing the client
ly extremely hungry because of increased
about dietary measures. The metabolism. The client should be instruct-
nurse tells the client that it
ed to consume a high-calorie diet with six
is important to eat foods that
full meals a day. The client should be in-
are: structed to eat foods that are nutritious and
contain ample amounts of protein, carbo-
a) high in bulk and fiber hydrates, fats, and minerals. Clients should
b) low in calories be discouraged from eating foods that in-
c) low in carbohydrates and crease peristalsis and thus result in diar-
fats rhea, such as highly seasoned, bulky, and
d) high in calories fibrous foods.

89. 17. A client with type 1 dia-17) A


betes mellitus tells the nurse
- It is important for clients with type 1 dia-
that mealtimes are not impor-betes mellitus to correlate eating with insulin
tant and that she eats when- administration to prevent hypoglycemia. In-
ever it is convenient. It is im-
sulin should be given at approximately the
portant for the nurse to ex- same time each day, and meals should be
plain that mealtimes: eaten at approximately the same time each
day. This will establish regular patterns of
a) must be approximately the glucose availability that approximate glu-
same time each day to main- cose availability in a nondiabetic body. Op-
tain a stable blood glucose tions B, C, and D are incorrect because they
b) can be varied as long as infer that mealtimes are not important.
the time of insulin adminis-
tration is also varied
c) are not important as long
as the client monitors the
blood glucose regularly
d) are not important as long
as snack foods are readily
available

34 / 45
Endocrine NCLEX questions
Study online at https://quizlet.com/_9oobst
90. 18. A client with type 1 dia- 18) B
betes mellitus tells the nurse, - An excess of insulin relative to the amount
"I usually begin to feel sick of blood glucose induces hypoglycemia. De-
late in the afternoon; is there pending on the length of action of the in-
something wrong with me?" sulin administered, the risk of hypoglycemia
The appropriate response by may be greatest in the late afternoon. The
the nurse is which of the fol- nurse needs to collect more data to deter-
lowing? mine if the client is actually experiencing
hypoglycemia. Asking the client to describe
a) don't worry about that. the sick feeling provides the nurse with more
Most diabetics feel that way data. Options A, C, and D are nontherapeu-
b) can you describe what you tic communication statements.
mean by feeling sick?
c) let me know if that happens
today
d) most people feel tired late
in the afternoon

91. 19. A nurse is gathering data 19) A


from a client newly diag- - Polydipsia, polyuria, and polyphagia are
nosed with diabetes mellitus the classic signs and symptoms of dia-
concerning events leading to betes mellitus. Dyspepsia, dysphagia, and
the client's seeking medical dysphasia are associated with other body
attention. The nurse identifies systems (gastric and neurological). Hyper-
which of the following as the glycemia also occurs.
major symptoms of diabetes
mellitus?

a) polydipsia, polyuria, and


polyphagia
b) dyspepsia, polyuria, and
polyphagia
c) hypoglycemia, polyuria,
and dysphagia
d) hypoglycemia, polyuria,
and dysphasia

92. 20. A husband of a client 20) A


with graves' disease express- - Frequently, family and friends may report

35 / 45
Endocrine NCLEX questions
Study online at https://quizlet.com/_9oobst
es concern regarding his that the client with Graves' disease has be-
wife's health because during come more irritable or depressed. The signs
the past 3 months she has and symptoms in the question are support-
been experiencing nervous- ing data for the nursing diagnosis of Inef-
ness, inability to concentrate fective coping and are not related to options
even on trivial tasks, and out- B, C, and D. The question does not provide
bursts of temper. On the ba- data to support options B, C, and D.
sis of this information, which
nursing diagnosis would the
nurse identify as appropriate
for the client?

a) ineffective coping
b) disturbed sensory percep-
tion
c) social isolation
d) grieving

93. 21. A nurse is caring for 21) D


a client with hypoparathy- - Medical management of hypoparathy-
roidism. In planning for dis- roidism is aimed at correcting the hypocal-
charge from the hospital, cemia. This is accomplished with prescribed
the nurse identifies which of medications as well as lifelong compliance
the following as a potential to dietary guidelines, which include con-
psychosocial nursing diag- sumption of foods high in calcium but low
nosis? in phosphorus. Knowing that the interven-
tions are lifelong can create some anxiety
a) impaired comfort related to for the client, and this problem needs to
cold intolerance secondary be addressed before hospital discharge. The
to decreased metabolic rate other options are unrelated to this condition
b) constipation related to and to a psychosocial concern.
decreased peristaltic ac-
tion secondary to decreased
metabolic rate
c) high risk for impaired skin
integrity related to edema
d) anxiety related to the need
for lifelong dietary interven-
tions to control the disease
36 / 45
Endocrine NCLEX questions
Study online at https://quizlet.com/_9oobst
94. 22. A nurse is caring for a 22) D
hospitalized older client with - The nurse should confirm that the client
a diagnosis of dehydration is hypoglycemic by checking the blood glu-
who also has diabetes melli- cose. Option A is incorrect because hypo-
tus. The client is alert but dis- glycemia has not been determined. More in-
oriented, pale, and slightly di- formation should be gathered before calling
aphoretic, and the nurse sus- the physician, so option B is incorrect. Op-
pects that the client is hypo- tion C does not meet the client's immediate
glycemic. The initial nursing needs.
intervention would be to:

a) administer oral glucose


b) assist the client to bed, put
the side rails up, and call the
physician
c) seat the client at the
nurse's desk while checking
the physician's order
d) obtain a fingerstick blood
specimen and test the glu-
cose level

95. 23. An adult client with di- 23) A


abetes mellitus reports to - The normal level for HgbA1C is 4.5% to
the health care clinic for a 7.5%. This test measures the amount of glu-
glycosylated hemoglobin A cose that has become permanently bound
(HgbA1c) level. Which labo- to the red blood cells from circulating glu-
ratory result indicates client cose. Elevations in blood glucose will cause
compliance with the pre- elevations in the amount of glycosylation. El-
scribed diabetic regimen? evations indicate continued need for teach-
ing related to prevention of hyperglycemic
a) 5% episodes.
b) 8%
c) 10%
d) 15%

96. 24. A client is diagnosed with 24) A


type 2 diabetes mellitus and - The client is denying the experience of a
is started on glyburide (Mi- chronic illness that will require her to make

37 / 45
Endocrine NCLEX questions
Study online at https://quizlet.com/_9oobst
cronase) 2.5 mg orally. The lifestyle changes. There is no evidence of
client smiles and says, "Oh, anger or depression in the statement made
good, as long as I take this by the client. The client has not accepted the
pill I can eat whatever I want." disease if expectations are unrealistic.
In this situation, the nurse's
intervention is focused on ad-
dressing which coping mech-
anism?

a) denial
b) anger
c) depression
d) acceptance

97. 25. A client newly diagnosed 25) A


with diabetes mellitus is ad- - Hyperglycemia is characteristic of new-
mitted to the hospital for eval- ly diagnosed diabetes mellitus. Newly di-
uation and control of the dis- agnosed diabetic clients present a variety
ease. When analyzing the as- of symptoms, which may include polydip-
sessment data, which of the sia, polyuria, polyphagia, weakness, weight
following would the nurse loss, and dehydration.
likely expect to find?

a) hyperglycemia
b) hypoglycemia
c) weight gain
d) hematuria

98. 26. A client with diabetes mel-26) B


litus says that it is very diffi-
- It is important to determine and deal with
cult to adhere to the diabetic a client's concerns and to identify measures
treatment plan. The nurse in- that will assist the client to comply with the
terprets the client's concern diabetic regimen. The nurse should deter-
and determines that the ap- mine if a knowledge deficit exists and if the
propriate response is: client's treatment plan maintains normalcy
as much as is possible with the lifestyle.
a) if you don't take your in- Scare tactics as described in options A and
sulin you will develop diabet- C should not be used. Positive reinforcement
ic ketoacidosis (DKA) is necessary instead of focusing on negative

38 / 45
Endocrine NCLEX questions
Study online at https://quizlet.com/_9oobst
b) let's go over your diet behaviors. Option D does not address the
again to be sure it contains subject of the question.
foods you like
c) do you understand what
noncompliance can mean to
your future health?
d) let's check your blood glu-
cose now

99. 27. A nurse provides instruc- 27) D


tions to a client who is sched- - The client undergoing a radioactive iodine
uled for a radioactive iodine uptake test needs to be reassured that the
uptake test. Which statement amount of radioactive iodine used is very
by the client indicates a need small, that it is not harmful to the client,
for further instructions? and that the client will not be radioactive.
The other options are correct regarding this
a) the test measures the rate diagnostic test.
of iodine uptake by my thy-
roid gland
b) I will need to drink a small
dose of radioactive iodine be-
fore the test
c) a 24 hour urine specimen
will need to be collected to
measure iodine excretion
d) I need to minimize close
contact with others in my
family for a period of 48 hours
after the test because of the
radioactivity in my system

100. 28. A nurse receives a report 28) C


that an adult client with deliri- - Blood glucose levels for an adult normally
um has a blood glucose level range between 60 and 120 mg/dL. A level of
of 33 mg/dL. The nurse ana- 33 mg/dL indicates hypoglycemia. Metabol-
lyzes this report as: ic disorders can be an etiological factor of
delirium.
a) higher than normal, indi-
cating a cause of the delirium

39 / 45
Endocrine NCLEX questions
Study online at https://quizlet.com/_9oobst
b) a normal reading for this
client
c) a lower than normal read-
ing, indicating a cause for the
delirium
d) insignificant and unrelated
to the delirium

101. 29. A nurse is caring 29) D


for a client with Cush- - Physical changes in the client's appear-
ing's syndrome who demon- ance can occur with Cushing's syndrome.
strates withdrawn behavior. Such changes include hirsutism, moon
The nurse recognizes that face, buffalo hump, acne, and striae. These
this client's behavior is likely changes cause a body image disturbance.
related to which nursing diag- Options A, B, and C are not commonly as-
nosis? sociated with Cushing's syndrome.

a) deficient diversional activ-


ity
b) powerlessness
c) hopelessness
d) disturbed body image

102. 30. A client with Addison's 30) D


disease makes all of the - The client with Addison's disease is expe-
following statements. Which riencing deficits of mineralocorticoids, glu-
one does the nurse analyze cocorticoids, and androgens. Aldosterone
as requiring further discus- deficiency affects the ability of the nephrons
sion? to conserve sodium, so the client experi-
ences sodium and fluid volume deficit. The
a) I wear a Medic-Alert client needs to manage this problem with
bracelet at all times daily hormone replacement and increased
b) I need to weigh myself dai- fluid and sodium intake. Clients are instruct-
ly and record it ed to weigh themselves daily as a means of
c) It is important that I drink monitoring fluid volume balance. Glucocor-
enough fluids and increase ticoids and mineralocorticoids are essential
my salt intake components of the stress response. Addi-
d) my medication doses will tional doses of hormone replacement ther-
apy are needed with any type of physical

40 / 45
Endocrine NCLEX questions
Study online at https://quizlet.com/_9oobst
not need to be adjusted for or psychological stressor. This information
any reason needs to be conveyed to the client and re-
quires that the client wear a Medic-Alert
bracelet so that health care professionals
are aware of this problem if the client were
to experience a medical emergency.

103. A client is admitted for re- Answer B is correct. A goiter is hyperplasia


moval of a goiter. Which of the thyroid gland. Removal of a goiter
nursing intervention should can result in laryngeal spasms and airway
receive priority during the occlusion. The other answers are lesser in
post-operative period? priority.
A. Maintaining fluid and elec-
trolyte balance
B. Assessing the client's air-
way
C. Providing needed nutrition
and fluids
D. Providing pain relief with
narcotic analgesics

104. A client is admitted for treat- Answer C is correct. The parathyroid is re-
ment of hypoparathyroidism. sponsible for calcium and phosphorus ab-
Based on the client's diagno- sorption. Clients with hypoparathyroidism
sis, the nurse would antici- have hypocalcemia. Answers A, B, and D
pate an order for: are not associated with hypoparathyroidism
A. Potassium therefore they are incorrect.
B. Magnesium
C. Calcium
D. Iron

105. A client with Addison's dis- Answer B is correct. Answer B is correct


ease will most likely exhibit because a bronze pigmentation is a sign of
which symptom? Addison's disease. Answers A, C, and D are
A. Hypertension symptoms of Cushing's syndrome, making
B. Bronze pigmentation them incorrect.
C. Hirsutism
D. Purple striae

106.
41 / 45
Endocrine NCLEX questions
Study online at https://quizlet.com/_9oobst
A client with Cushing's syn- Answer B is correct. A client with Cushing's
drome should be instructed syndrome has adrenocortical hypersecre-
to: tion, so she retains sodium and water. The
A. Avoid alcoholic beverages client may drink alcohol in moderation, so
B. Limit the sodium in her answer A is incorrect, and there is no need
diet to eat more green vegetables or limit protein,
C. Increase servings of dark so answers C and D are incorrect.
green vegetables
D. Limit the amount of pro-
tein in her diet

107. The client with a suspected Answer A is correct. The pituitary is located
pituitary tumor will most like- in the middle of the skull adjacent to the
ly exhibit symptoms of: optic nerve and brain. Pressure on the optic
A. Alteration in visual acuity nerve can cause an increase in intracra-
B. Frequent diarrhea nial pressure. Clients frequently complain of
C. Alterations in blood glu- headache, nausea, vomiting, and decreas-
cose ing visual acuity as the intracranial pressure
D. Urticaria increases. B, C, and D are incorrect because
they are not associated with a pituitary tu-
mor.

108. A diabetic client has been Answer A is correct. Glucophage (met-


maintained on Glucophage formin) can cause renal complications. The
(metformin) for regulation of client should be monitored for changes in re-
his blood glucose levels. nal function. In answer B, the medication be-
Which teaching should be in- gins working immediately, so it is incorrect.
cluded in the plan of care? In answer C, the amount of carbohydrates
A. Report changes in urinary should be regulated with a diabetic diet, so
pattern. it is incorrect. The use of lotions in answer D
B. Allow six weeks for opti- is unnecessary, so it is incorrect.
mal effects.
C. Increase the amount of
carbohydrates in your diet.
D. Use lotions to treat itching.

109. A client with diabetes ex- Answer D is correct. Somogyi's is charac-


periences Somogyi's effect. terized by a drop in glucose levels at ap-
To prevent this complication, proximately 2 a.m. or 3 a.m. followed by a

42 / 45
Endocrine NCLEX questions
Study online at https://quizlet.com/_9oobst
the nurse should instruct the false elevation. Eating a protein and carbo-
client to: hydrate snack before retiring prevents the
A. Take his insulin each day hypoglycemia and rebound elevation. An-
at 1400 hours swers A, B, and C are incorrect because
B. Engage in physical activity they do not prevent Somogyi's effect.
daily
C. Increase the amount of
regular insulin
D. Eat a protein and carbohy-
drate snack at bedtime

110. Which item should be kept at Answer D is correct. Laryngeal swelling is


the bedside of a client who not uncommon in clients following a thy-
has just returned from having roidectomy. A tracheostomy tray should be
a thyroidectomy? kept available. The ventilator is not neces-
A. A padded tongue sary, so answer A is incorrect. The endotra-
B. An endotracheal tube cheal tube is very difficult, if not impossible,
C. An airway to intubate if swelling has already occurred,
D. A tracheostomy set so answer B is incorrect. The airway will
do no good because the swelling is in the
trachea, so answer C is incorrect.

111. Which vitamin is directly in- Answer B is correct. Vitamin D is related to


volved in the metabolism of absorption of calcium and phosphorus. A,
the hormones secreted by C, and D are incorrect because they are
the parathyroid? not related to the absorption of calcium and
A. Vitamin C phosphorus.
B. Vitamin D
C. Vitamin K
D. Vitamin B9

112. A client with acromegaly will Answer A is correct. Acromegaly is an in-


most likely experience which crease in secretion of growth hormone.
symptom? The growth hormones cause expansion
A. Bone pain and elongation of the bones. Answers B,
B. Frequent infections C, and D are not directly associated with
C. Fatigue acromegaly, so they are incorrect.
D. Weight loss

113.
43 / 45
Endocrine NCLEX questions
Study online at https://quizlet.com/_9oobst
A diabetic client is taking Answer B is correct. This insulin, unlike oth-
Lantus insulin for regulation ers, is most frequently administered at night.
of his blood glucose levels. Its duration is 24-36 hours. A, C, and D are
The nurse should know that incorrect they are incorrect times to admin-
this insulin will most likely be ister Lantus insulin.
administered:
A. Prior to each meal
B. At night
C. Midday
D. Prior to the evening meal

114. A client with polyuria, poly- Answer B is correct. The client with hy-
dipsia, and polyphagia is di- perglycemia will exhibit polyuria, polydipsia,
agnosed with diabetes melli- or increased thirst, and polyphagia, or in-
tus. The nurse would expect creased hunger. A, C, and D are incorrect
that these symptoms are re- because they are not signs of hypoglycemia.
lated to
A. Hypoglycemia
B. Hyperglycemia
C. Hyperparathyroidism
D. Hyperthyroidism

115. Which laboratory test con- Answer C is correct. The Hgb A-1C indi-
ducted on the client with dia- cates that the client has been compliant for
betes mellitus indicates com- approximately three months. Answers A, B,
pliance? and D tell the nurse the client's blood glu-
A. Fasting blood glucose cose at the time of the test, so they are
B. Two-hour post-prandial incorrect.
C. Hgb A-1C
D. Dextrostix

116. Acromegaly is most frequent- 1. A: Acromegaly results from benign tumors


ly diagnosed in: on the pituitary gland that produce exces-
a. Middle-aged adults sive amounts of growth hormone. Although
b. Newborns symptoms may present at any age, the
c. Children ages 2 to 5 diagnosis generally occurs in middle-aged
d. Adults age 65 and older persons. Untreated, the consequences of
acromegaly include type 2 diabetes, hyper-

44 / 45
Endocrine NCLEX questions
Study online at https://quizlet.com/_9oobst
tension and increased risk of cardiovascular
disease, arthritis and colon polyps.

45 / 45

You might also like